You are on page 1of 56

Pediatrics 01May2009

Pediatrics #1 – Clinical: Part One


1) What is the most common etiology of pediatric cardiac arrest?
a) Respiratory
b) Cardiac
c) CNS
d) Metabolic
e) Renal
2) When assessing a pediatric patient in cardiorespiratory arrest, the primary survey
involves ABCDE (airway, breathing, circulation, disability, and exposure). Which of the
following components of the primary survey is contraindicated in children?
a) Head-tilt chin-lift
b) Jaw-thrust maneuver
c) Blind finger sweep
d) Using heart rate to assess intravascular volume status
e) Using capillary refill to assess adequate circulation
3) Which of the following is a primary treatment option for a pediatric patient in
pulseless ventricular tachycardia (VT) or ventricular fibrillation (VF)?
a) Cardioversion
b) Amiodarone or procainamide
c) Vagal maneuvers and adenosine
d) Atropine and epinephrine
e) Defibrillation
4) A child presents with a cardiac arrhythmia. The Emergency Department physician puts
a bag of ice-cold water on the child’s face to induce the mammalian diving reflex (vagal
maneuver). Which of the following did the patient likely have?
a) Hemodynamically stable VT
b) Hemodynamically stable SVT
c) Hemodynamically unstable VT or SVT
d) Pulseless VT or VF
e) Bradycardia or asystole
5) Which of the following drugs would be useful for multiple ventricular ectopy (PVCs)
as well as for refractory VT/VF to improve the susceptibility for cardioversion?
a) Atropine
b) Bicarbonate
c) Calcium and magnesium
d) Lidocaine
e) Epinephrine
6) What is the most common type of shock seen in children?
a) Hypovolemic (electrolyte loss, blood loss, third spacing)
b) Cardiogenic (congenital heart disease, cardiomyopathy)
c) Disruptive (anaphylactic, neurologic, drug toxicity)
d) Septic, compensated (“warm”)
e) Septic, uncompensated (“cold”)
7) Approximately what percentage of childhood poisonings occur in children younger
than 5 years of age?
a) 15%

DO NOT DISTRIBUTE -1-


Pediatrics 01May2009

b) 35%
c) 50%
d) 65%
e) 85%
8) A child presents with irritability, anorexia, vomiting, and hyperactivity. The clinician
suspects lead poisoning and draws labs looking for >20mcg/dL, but the labs return
>45mcg/dL so the clinician immediately begins chelation treatment. What is the most
likely source of this child’s poisoning?
a) Playground equipment
b) Paint from an old building
c) Men’s multivitamin tablets
d) Window blinds
e) Graphite pencils
9) What is the leading cause of accidental death in children older than 1 year and
adolescents?
a) Drowning
b) Lead poisoning
c) Burns
d) Motor vehicle accidents
e) Foreign body aspiration
10) Which of the following pediatric patients is most likely to be involved in a drowning
accident (bathtub submersion)?
a) Caucasian males
b) Caucasian females
c) African American males
d) African American females
11) Foreign body aspiration has the highest incidence in children aged 6-30 months.
What is the most common object involved, accounting for over 50% of aspirations?
a) Small toys (e.g. lego, marble)
b) Nuts
c) Bread
d) Crayons
e) Fish bones
12) Burns are the 3rd leading cause of injury in children and the 2nd most frequent cause
of accidental death. They involve abuse in 15-25% of cases. What is the most common
type of burn seen in children?
a) Scald burns
b) Flame burns
c) Electrical burns
d) Contact burns
e) Chemical burns
13) Which of the following is NOT true of child abuse?
a) Neglect is the most common cause of failure to thrive
b) Sexual abusers of children are usually stepfathers, fathers, or other male family
members
c) 10% of ER visits involving children younger than 5 years old result from abuse

DO NOT DISTRIBUTE -2-


Pediatrics 01May2009

d) 80% of sexual abuse victims are girls


e) Molestation by strangers and babysitters is common
14) It has been shown that home apnea monitors do not decrease the likelihood of sudden
infant death syndrome (SIDS). Which of the following is NOT a risk factor for SIDS?
a) Prematurity
b) Low birth weight
c) Female gender
d) Prone sleeping position
e) Intrauterine growth restriction (IUGR)
f) Maternal smoking during pregnancy
15) Newborn cyanosis is most likely due to cardiac, pulmonary, neurologic, or
hematologic disorders. It is clinically evident when absolute concentrations of
deoxygenated hemoglobin rise above:
a) 1g/dL
b) 2g/dL
c) 3g/dL
d) 4g/dL
e) 5g/dL
16) Which of the following should be given to an unstable infant with suspected
congenital heart disease?
a) Prostaglandin E1 (alprostadil)
b) Prostaglandin E2 (dinoprostone)
c) Phosphodiesterase 3 inhibitor (milrinone)
d) Phosphodiesterase 4 inhibitor (mesembrine)
e) Phosphodiesterase 5 inhibitor (sildenafil)
17) During a routine cardiac examination of an infant, a murmur is heard. Further
examination reveals a fixed split S2, regardless of inspiration or expiration. An ECG
shows mild right ventricular hypertrophy (RVH). Which of the following is most likely?
a) Atrial septal defect
b) Ventricular septal defect
c) Patent ductus
d) Tetrology of Fallot
e) Coarctation of the aorta
f) Transposition of the great arteries
18) Which of the following presents with a holosystolic murmur and is the most common
congenital heart defect?
a) Patent ductus
b) Atrial septal defect
c) Tetrology of Fallot
d) Coarctation of the aorta
e) Ventricular septal defect
f) Transposition of the great arteries
19) A continuous “machinery” murmur is heart in which of the following?
a) Patent ductus
b) Atrial septal defect
c) Tetrology of Fallot

DO NOT DISTRIBUTE -3-


Pediatrics 01May2009

d) Coarctation of the aorta


e) Ventricular septal defect
f) Transposition of the great arteries
20) A young child presents with cyanosis. The mother says the child has periodic
episodic cyanosis, which is accompanied by agitation and rapid, deep breathing. The
child turns blue for a couple minutes then normal color returns. Chest x-ray shows a
“boot shaped” heart and ECG shows right-axis deviation. Which of the following would
NOT be seen in this child?
a) Over-riding aorta
b) Atrial septal defect
c) Right ventricular hypertrophy
d) Pulmonary stenosis
21) A newborn is found to have Turner syndrome. Blood pressure measurements show a
difference between upper and lower extremities. Which of the following is most likely?
a) Patent ductus
b) Atrial septal defect
c) Tetrology of Fallot
d) Coarctation of the aorta
e) Ventricular septal defect
f) Transposition of the great arteries
22) A newborn boy presents to the Emergency Room with cyanosis. A chest x-ray shows
an egg-on-a-string (egg-shaped) heart. Which of the following is most likely?
a) Patent ductus
b) Atrial septal defect
c) Tetrology of Fallot
d) Coarctation of the aorta
e) Ventricular septal defect
f) Transposition of the great arteries
23) Conotruncal anomalies (e.g. truncus arteriosus, tetralogy of Fallot, VSD) are
associated with what chromosomal micro-deletion?
a) 11p22
b) 11q22
c) 22p11
d) 22q11
24) What is the most common cardiac lesion found in rheumatic heart disease?
a) Hypoplastic left heart syndrome
b) Coronary artery aneurysms
c) Aortic stenosis
d) Mitral regurgitation
e) Ventricular septal defect
25) A young child is found to have pericarditis, myocarditis, and transient rhythm
disturbances. If Kawasaki disease is suspected, which of the following may develop?
a) Hypoplastic left heart syndrome
b) Coronary artery aneurysms
c) Aortic stenosis
d) Mitral regurgitation

DO NOT DISTRIBUTE -4-


Pediatrics 01May2009

e) Ventricular septal defect


26) Endocarditis is seen in adult IV drug abusers and with prosthetic replacement valves
after dental procedures. In children, it is seen with congenitally abnormal valves. What is
the most common causative agent?
a) Staphylococcus aureus
b) Streptococcus viridans
c) Haemophilus influenzae (HACEK)
d) Staphylococcus epidermidis
e) Coxsackie B virus
27) A child presents with fever, dyspnea, tachycardia, and mild ST-and T-wave changes
on ECG. Physical exam reveals S3 ventricular gallop. If myocarditis is suspected, which
of the following is most likely?
a) Staphylococcus aureus
b) Streptococcus viridans
c) Haemophilus influenzae (HACEK)
d) Staphylococcus epidermidis
e) Coxsackie B virus
28) Dyspnea, orthopnea, paroxysmal nocturnal dyspnea (PND), pulsus alternans, and
neuromuscular disease etiology (e.g. Duchenne Muscular Dystrophy) is seen in which of
the following?
a) Hypertrophic cardiomyopathy
b) Dilated cardiomyopathy
c) Endocarditis
d) Myocarditis
e) Myocardial infarction
29) A young high school basketball player suddenly collapses during a game. Which of
the following is most likely?
a) Hypertrophic cardiomyopathy
b) Dilated cardiomyopathy
c) Endocarditis
d) Myocarditis
e) Myocardial infarction
30) Which of the following developmental tests evaluates children from 0 to 6 years of
age and involves gross motor, fine motor-adaptive, language, and personal-social?
a) Denver II
b) CAT
c) CLAMS
d) B & C
31) What is the best indicator of future intellectual achievement?
a) Gross motor
b) Fine motor-adaptive
c) Language
d) Personal-social
32) A development quotient below what level would constitute developmental delay?
a) 100
b) 90

DO NOT DISTRIBUTE -5-


Pediatrics 01May2009

c) 80
d) 70
e) 60
33) Which of the following intelligence quotients (IQs) would signify severely mentally
retarded?
a) 80
b) 60
c) 45
d) 30
e) 10
34) Which of the following is NOT true of attention deficit hyperactivity disorder
(ADHD)?
a) Most of those affected as a child do not have persisting symptoms
b) Involves hyperactivity, inattention, and impulsivity
c) Requires information from parents and teachers using a scale such as the
Conner Parent and Teacher Scales
d) Symptoms must be present for at least 6 months
e) Treatment may include drugs such as methylphenidate (Ritalin),
dextroamphetamine (Dexedrine) and mixed amphetamine salts (Adderall)
35) A child is able to balance on one foot, can copy a cross, can catch a ball, can dress
himself, and tells an intelligible story to a stranger. How old is this child?
a) 1-year
b) 2-years
c) 3-years
d) 4-years
e) 5-years
36) A child is able to discriminate the use of “mama” and “dada”, stands alone,
cooperates with dressing, and imitates actions. How old is this child?
a) 36-months
b) 24-months
c) 12-months
d) 6-months
e) 2-months
37) A child can roll in both directions, sits still, reaches with one hand, and babbles. How
old is this child?
a) 2-months
b) 4-months
c) 6-months
d) 9-months
e) 12-months
38) Autism and Asperger syndrome are pervasive development disorders (PDD).
Asperger is characterized by intense interest in specific topics (e.g. dinosaurs) as well as
which of the following?
a) Lack of language ability
b) Lack of showing interest
c) Lack of emotional reciprocity

DO NOT DISTRIBUTE -6-


Pediatrics 01May2009

d) Lack of abstract form understanding (e.g. sarcasm)


e) Lack of motor mannerisms and inflexibility to rituals
39) The measles-mumps-rubella (MMR) vaccine and thimerosal have been shown to be
associated with the development of autism.
a) True
b) False
Match the disease with the causative agent:
40) Measles a) Parvovirus B19
41) Rubella b) Paramyxovirus
42) Roseola infantum c) Varicella-Zoster virus
43) Fifth disease (erythema infectiosum) d) RNA Togavirus
44) Hand-Foot-and-Mouth e) HHV-6
45) Chicken pox f) Coxsackie A virus
Match the disease with the type of rash:
46) Measles a) Mouth ulcers, football shaped vesicles
47) Rubella b) Pruritic rash, red papules, vesicles, scabs
48) Roseola infantum c) Maculopapular on trunk to periphery
49) Fifth disease (erythema infectiosum) d) Discrete maculopapular, <5 days
50) Hand-Foot-and-Mouth e) Koplik spots, maculopapular at head
51) Chicken pox f) Slapped-cheek, arms spread to trunk
52) An infant presents with abrupt onset of diffuse erythema, marked skin tenderness,
and fever. Flaccid bullae develop and rupture, leading beefy red, weeping surfaces.
Physical exam reveals separation of the epidermis on light rubbing (Nikolsky sign). Labs
show S. aureus as the causative agent. Which of the following is most likely?
a) Bullous impetigo
b) Nonbullous impetigo
c) Scalded skin syndrome
d) Folliculitis
e) Cellulitis
53) A child presents with a hyperpigmented scaly lesion on the upper back, chest, and
neck. Labs are negative for septated branching hyphae. However, a “spaghetti and
meatballs” arrangement is seen on microscopy. Further lab testing shows Malassezia
furfur. Which of the following is most likely?
a) Tinea capitis
b) Tinea corporis
c) Tinea cruris
d) Tinea pedis
e) Tinea versicolor
54) A boy with oily skin presents with complaints of facial blemishes that began as he
reached puberty. The clinician believes Propionibacterium acnes is to blame. What
treatment option is reserved for severe cases that involve inflammatory papules, cysts,
abscesses, and scaring?
a) Benzoyl peroxide
b) Topical antibiotics (clindamycin)
c) Topical retinoids (tretinoin)
d) Oral antibiotics (tetracycline)

DO NOT DISTRIBUTE -7-


Pediatrics 01May2009

e) Oral retinoic acid (Accutane)


55) A child is found to have a silvery scale rash on areas of physical or thermal trauma
(Köbner phenomenon). The child scratches at the rash, which causes pinpoint bleeding
(Auspitz sign). Which of the following is most likely?
a) Acne
b) Tinea corporis
c) Folliculitis
d) Psoriasis
e) Bullous impetigo
56) Which of the following is NOT characteristic of diabetic ketoacidosis?
a) Type II diabetes
b) Polyuria, polydipsia
c) Kussmaul breathing
d) Tachycardia and hypotension
e) Fruity breath and cerebral edema
57) An infant girl is found to have short statue and deformities of the neck, chest, and
heart. Physical exam reveals a knuckle-knuckle-dimple-dimple sign. Which of the
following is most likely?
a) Central diabetes insipidus
b) Growth hormone deficiency
c) Primary hypothyroidism
d) Cushing syndrome
e) Turner syndrome
58) What is the most common cause of hyperthyroidism in children?
a) Hashimoto thyroiditis
b) Graves disease
c) Toxic multinodular goiter
d) Amiodarone toxicity
e) Knuckle-knuckle-dimple-knuckle pseudohypothyroidism
59) What is the most common cause of congenital adrenal hyperplasia?
a) 21-hydroxylase deficiency
b) 17alpha-hydroxylase deficiency
c) Aromatase deficiency
d) 11beta-hydroxylase deficiency
e) 3beta-hydroxylase deficiency
60) Precocious puberty is defined as development of secondary sex characteristics before
age 7.5 in girls and before what age in boys?
a) 5
b) 6
c) 7.5
d) 9
e) 10
61) A child presents with weakness, nausea, vomiting, and weight loss. History reveals
salt cravings. Physical exam reveals increased skin pigmentation and postural
hypotension. Lab testing shows hyponatremia, hyperkalemia, hypoglycemia, and
metabolic acidosis. Which of the following is most likely?

DO NOT DISTRIBUTE -8-


Pediatrics 01May2009

a) Cushing syndrome
b) Cushing disease
c) Addison disease
d) Pubertal delay
e) Pseudopseudohypothyroidism
62) What is the Holliday-Segar method used for?
a) Fluid bolus calculation in burn victims
b) Fluid infusion rate for traumatic injuries
c) Fluid amount for daily maintenance
d) Fluid allowance for diabetes insipidus
e) Fluid loss rate for dehydration
63) A child presents with suspected dehydration due to immature kidneys. Evaluation
shows a depressed anterior fontanelle, sunken eyes, altered mental status, and increased
heart rate. It is estimated that the child has lost 7% of their body weight to water. Labs
reveal 800mOsm/L urine, slightly elevated urine specific gravity, elevated BUN, and
mild acidosis. Which of the following best describes this patient?
a) Normal
b) Mild shock
c) Compensated shock
d) Uncompensated shock
64) The hemolysis of red blood cells during sample collections is the most common
causes of (artifactual):
a) Hyponatremia
b) Hypernatremia
c) Hypokalemia
d) Hyperkalemia
65) A child presents with constipation after a bout of severe vomiting. The clinician notes
weakness and tetany. An ECG shows depressed ST segments with biphasic T-waves and
prominent U-waves. Which of the following is most likely?
a) Hyponatremia
b) Hypernatremia
c) Hypokalemia
d) Hyperkalemia
66) A child with a congenital renal tubule defect has large losses of bicarbonate. Labs
reveal a blood pH of 7.20. Which of the following is most likely?
a) Metabolic acidosis
b) Metabolic alkalosis
c) Respiratory acidosis
d) Respiratory alkalosis

Pediatrics #2 – Clinical: Part Two


1) What is the most common medical cause of abdominal pain in children?
a) Sickle cell disease
b) Lactase deficiency
c) Gastroenteritis
d) Mesenteric lymphadenitis

DO NOT DISTRIBUTE -9-


Pediatrics 01May2009

e) Group A strep infections


2) What is the most common surgical cause of abdominal pain in children?
a) Trauma
b) Intussusception
c) Cholecystitis
d) Appendicitis
e) Testicular torsion
3) Which of the following cases would suggest colicky pain or obstruction?
a) Blood or mucinous diarrhea
b) Child lies still
c) Child cannot remain still
d) Sore throat
e) Dysuria
4) Which of the following cases would suggest inflammatory pain, infection, or
perforated organ/viscus?
a) Blood or mucinous diarrhea
b) Child lies still
c) Child cannot remain still
d) Sore throat
e) Dysuria
5) An infant presents with violent episodes of irritability, colicky pain, and emesis that
are interspersed with relatively normal periods. A barium enema is performed, showing a
coiled-spring appearance to the bowel. Which of the following is most likely?
a) Trauma
b) Intussusception
c) Cholecystitis
d) Appendicitis
e) Testicular torsion
6) Which of the following is an uncommon cause of emesis in children?
a) Crohn disease
b) Gastroesophageal reflux
c) Acute gastroenteritis
d) Tonsillitis
e) Otitis media
7) A three-week-old child presents with non-bilious projectile vomiting. Which of the
following is most likely?
a) Gastroesophageal reflux
b) GI malrotation
c) Pyloric stenosis
d) Hirschsprung disease
e) Esophageal stricture
8) A newborn is brought in with gastrointestinal problems. An upper GI series is
performed, showing abnormal positioning of the ligament of Treitz and the cecum.
Which of the following is most likely?
a) Gastroesophageal reflux
b) GI malrotation

DO NOT DISTRIBUTE - 10 -
Pediatrics 01May2009

c) Pyloric stenosis
d) Hirschsprung disease
e) Esophageal stricture
9) What is the most common cause of acute diarrhea in children within most developed
countries?
a) Bacterial enterocolitis
b) Appendicitis
c) Iron, mercury, lead, or fluoride ingestion
d) Intussusception
e) Viral gastroenteritis
10) Hirschsprung disease (i.e. congenital megacolon) results from failure of ganglion
cells of the myenteric plexus to migrate down to the distal colon. This results in:
a) Constipation
b) Diarrhea
c) Hematochezia
d) Obstipation
11) Which of the following is defined as bright red blood per rectum (BRBPR) and
usually results from a lower GI bleed, distal to the ligament of Treitz?
a) Constipation
b) Diarrhea
c) Hematochezia
d) Obstipation
e) Melena
12) Which of the following is the most common anomaly of the GI tract, is due to
vestigial remnant of the omphalomesenteric duct (within 100cm of the ileocecal valve),
and presents with painless rectal bleeding?
a) Gastroschisis
b) Meckel diverticulum
c) Hirschsprung disease
d) Omphalocele
e) Diaphragmatic hernia
13) Which of the following is NOT true of congenital defects?
a) Autosomal dominant disorders typically code for structural proteins
b) Autosomal recessive disorders typically code for enzymes
c) Environmental factors cause 10% of birth defects
d) Major anomalies have a low incidence up to newborns with the presence of
three minor anomalies
e) Ornithine transcarbamylase deficiency (OTC), like all other inborn errors of
metabolism, is autosomal recessive
Match the genetic defect with the characteristics:
14) Horseshoe kidney, rocker-bottom feet, low-set ears a) Downs (21)
15) Gynecomastia, small phallus, small testes, taller b) Edwards (18)
16) Wide-spaced hypoplastic nipples, webbed neck, shield chest c) Patau (13)
17) High leukemia risk, short, broad hands, dysmorphic face d) Turner
18) Microcephaly, sloped forehead, cryptorchidism, polydactyly e) Klinefelter

DO NOT DISTRIBUTE - 11 -
Pediatrics 01May2009

19) A mother brings in her young boy because he is gaining weight rapidly. She has been
trying to keep food away from him, but finds him digging through the garbage and eating
non-edible items (pica). Physical exam reveals almond-shaped eyes that are different
from the mother’s, a down-turned mouth, and small hands and feet. Which of the
following is most likely?
a) Maternal chromosome 15 deletion
b) Paternal chromosome 15 deletion
c) Trisomy 21
d) Trisomy 18
e) Trisomy 13
20) A girl is brought in by her mother with the concern that the child is not learning
properly. Physical exam reveals maxillary hypoplasia, a large mouth, prognathism, and
short stature. Mental assessment reveals the child is severely mentally retarded. During a
discussion with the mother, the clinician notes the child making jerky arm movements,
marionette-like movements, and laughing a great deal (happy puppet syndrome). Which
of the following is most likely?
a) Maternal chromosome 15 deletion
b) Paternal chromosome 15 deletion
c) Deletion of 22q11
d) Deletion of 22p11
e) Trisomy 21
21) A newborn is found to have macrosomia at birth. Testing as the child begins to grow
reveals macroorchidism, large ears, and a large jaw. Genetic testing reveals a large
amount of CGG trinucleotide repeats. Which of the following is most likely?
a) Angelman syndrome
b) Prader-Willi syndrome
c) Fragile X syndrome
d) DiGeorge syndrome
e) Hurler syndrome
22) Microdeletion of 22q11.2 can lead to tetralogy of Fallot, interrupted aortic arch, and
thymus absence. It is associated with velocardiofacial syndrome as well as:
a) Angelman syndrome
b) Prader-Willi syndrome
c) Fragile X syndrome
d) DiGeorge syndrome
e) Hurler syndrome
23) A 2-week-old presents with vision problems, anorexia, and emesis after breast-
feeding. Testing reveals liver failure, renal dysfunction, and developing cataracts. A
deficiency in G1P uridyltransferase is found. The clinician is concerned about the risk of
E. coli sepsis. Which of the following is most likely?
a) OTC deficiency
b) Gaucher disease
c) Hurler syndrome
d) Phenylketonuria
e) Galactosemia
f) Homocystinuria

DO NOT DISTRIBUTE - 12 -
Pediatrics 01May2009

Match the glycogen storage disease with the deficiency:


24) Acid alpha-glucosidase (GAA) a) Von Gierke disease (Type I)
25) Glucose-6-phosphate (G6P) b) Pompe disease (Type II)
26) Myophosphorylase c) McArdle disease (Type V)
27) A fair-haired, fair-skinned, blue-eyed child presents with projectile vomiting, mental
retardation, tremors, and mouse-like smelling urine. Testing reveals a defect in tyrosine
formation. Which of the following is most likely?
a) OTC deficiency
b) Gaucher disease
c) Hurler syndrome
d) Phenylketonuria
e) Galactosemia
f) Homocystinuria
28) Which of the following can cause eye lens dislocation and involves a defect in the
pathway that converts methionine to cysteine and serine?
a) OTC deficiency
b) Gaucher disease
c) Hurler syndrome
d) Phenylketonuria
e) Galactosemia
f) Homocystinuria
29) A patient is diagnosed with OTC deficiency after developing severe
hyperammonemia after the consumption of proteins. How is this disorder inherited?
a) Autosomal dominant
b) Autosomal recessive
c) X-linked
d) Y-linked
e) Mitochondrial
30) Which of the following is due to a deficiency in alpha-iduroindase leading to
dermatan and heparan sulfate accumulation, coarse facies, corneal clouding, and
exaggerated kyphosis?
a) OTC deficiency
b) Gaucher disease
c) Hurler syndrome
d) Phenylketonuria
e) Galactosemia
f) Homocystinuria
31) Which of the following is due to a deficiency in beta-glucosidase leading to
glucocerebrosidase accumulation, anemia, leucopenia, and an Erlenmeyer flask-shape of
the distal femur?
a) OTC deficiency
b) Gaucher disease
c) Hurler syndrome
d) Phenylketonuria
e) Galactosemia
f) Homocystinuria

DO NOT DISTRIBUTE - 13 -
Pediatrics 01May2009

32) An infant presents with anorexia, apathy, easy fatigability, and irritability. Physical
exam reveals skin pallor, glossitis, and koilonychias. Labs reveal iron deficiency. Which
of the following is most likely?
a) Hypochromic microcytic anemia
b) Hyperchromic microcytic anemia
c) Hypochromic macrocytic anemia
d) Hyperchromic macrocytic anemia
e) Normocytic anemia
33) During the first year of life, a child develops severe hemolytic anemia and
splenomegaly. A physical exam reveals a tower skull, frontal bossing, and prominent
cheekbones. Labs are negative for excess beta-globin tetramers. The child is started on
folate supplementation, RBC transfusions, and iron chelation. Which of the following is
most likely?
a) Homozygous alpha thalassemia (Bart)
b) Hemoglobin H disease
c) Alpha thalassemia minor
d) Beta thalassemia major
e) Beta thalassemia minor
34) A 2-year-old develops bone marrow suppression after a viral infection. Pure red cell
aplasia is found and the child is diagnosed with transient erythroblastopenia of childhood
(TEC). The mother is told it will pass. What type of anemia is this?
a) Hypochromic microcytic anemia
b) Hyperchromic microcytic anemia
c) Hypochromic macrocytic anemia
d) Hyperchromic macrocytic anemia
e) Normocytic anemia
35) An African-American child presents with pain in the hands, knees, and a general
aching sensation. Physical exam reveals splenomegaly, dactylitis, and pulmonary rales.
Labs show anemia. The father has a history of some type of RBC disorder. Which of the
following does the child most likely have?
a) Beta-globin position 4 substitution of glutamine for valine
b) Beta-globin position 4 substitution of valine for glutamine
c) Beta-globin position 6 substitution of glutamine for valine
d) Beta-globin position 6 substitution of valine for glutamine
36) A blood test on a child reveals Heinz bodies. Further testing reveals G6PD, which is a
lack of the hexose monophosphate shunt pathway enzyme that results in a depletion of
NADPH and inability to regenerate reduced glutathione. How is this disorder inherited?
a) Autosomal dominant
b) Autosomal recessive
c) X-linked
d) Y-linked
e) Mitochondrial
37) A child is brought to the emergency center with traumatic hemorrhages after a soccer
game. Physical exam reveals significant knee hemarthroses. Further testing reveals an X-
linked recessive disorder involving factor VIII deficiency. Which of the following is most
likely?

DO NOT DISTRIBUTE - 14 -
Pediatrics 01May2009

a) Sickle cell disease


b) Alpha thalassemia
c) Hemophilia A
d) Hemophilia B
e) Von Willebrand disease
38) A child presents with complaints of bleeding gums when she brushes her teeth and
even when she eats. Testing reveals a deficiency in von Willebrand factor. What is the
drug of choice for this patient for bleeding episodes?
a) Vitamin K
b) Clopidogrel
c) Desmopressin acetate
d) Cryoprecipitate
e) Fresh-frozen plasma
39) What clotting factor is deficient in hemophilia B?
a) Factor V
b) Factor VII
c) Factor VIII
d) Factor IX
e) Factor X
40) Which of the following is NOT true?
a) Hypocalcemic tetany with absence of the thymic shadow and cell-mediated
immunodeficiency suggests 22q11 deletion
b) Humoral immunodeficiency predisposes patients to infections with
encapsulated organisms (e.g. H. influenzae, S. pneumonia)
c) Cell-mediated immunodeficiency predisposes patients to autoimmune
disorders, intracellular organisms, and opportunistic infections
d) Typical signs of infections are often absent in the presence of neutropenia
e) The most common immunodeficiency seen in pediatrics is cell-mediated
41) A patient presents with recurrent infections involving catalase-producing bacteria (S.
aureus, C. albicans, Aspergillus). A nitroblue tetrazolium test (NBT) and
dihydrorhodamine reduction (DHR) test are positive for the detection of chronic
granulomatous disease (CGD). Along with interferon-gamma, what prophylactic daily
drug should this patient receive?
a) Penicillin G
b) Amphotericin B
c) Doxycycline
d) TMP-SMX
e) Isoniazid
42) A child presents with seasonal rhinorrhea and upper respiratory tract symptoms. He
has dark circles under his eyes and a horizontal crease across the middle of the nose. A
Type I hypersensitivity response is suspected. What is the treatment of choice?
a) Sedating H1 blockers (e.g. diphenhydramine)
b) Non-sedating H1 blockers (e.g. fexofenadine)
c) H2 blocker (e.g. famotidine)
d) Corticosteroid (e.g. prednisone)
e) Antibiotic (e.g. amoxicillin)

DO NOT DISTRIBUTE - 15 -
Pediatrics 01May2009

43) What is the gold standard for diagnosis of food allergies, such as to peanuts, eggs,
milk, soy, wheat, or fish?
a) Patient history
b) Allergy scratch test
c) Allergy blood panel
d) H1 suppression test
e) Double-blind placebo-controlled food challenge
44) A 13-year-old girl presents with joint pain lasting for 2-months. She admits to joint
stiffness in the morning that resolves with movement. Her mother says she has been
eating less, fatigues easier, and is irritable. Physical exam reveals a rash and limited range
of motion in the affected joints. Which of the following is most likely?
a) Systemic lupus erythematosus (SLE)
b) Polyarteritis nodosa (PAN)
c) Kawasaki disease
d) Juvenile rheumatoid arthritis (JRA)
e) DiGeorge syndrome
45) A late adolescent presents with a malar facial rash, photosensitivity, and painless oral
ulcerations. Testing reveals anemia, positive ANA, and signs of a type III
hypersensitivity reaction. What is the treatment of choice?
a) Sedating H1 blockers (e.g. diphenhydramine)
b) Non-sedating H1 blockers (e.g. fexofenadine)
c) H2 blocker (e.g. famotidine)
d) Corticosteroid (e.g. prednisone)
e) Antibiotic (e.g. amoxicillin)
46) PAN usually presents with waxing and waning symptoms of painful erythematous
skin nodules, purpura, hypertension, hematuria, and systemic complaints. Henoch-
Schönlein purpura is an IgA-mediated vasculitis that peaks in winter months, is usually
preceded by a group A streptococcal upper respiratory infection, and causes non-
thrombocytopenic palpable purura. What is the treatment of choice for both of these?
a) Sedating H1 blockers (e.g. diphenhydramine)
b) Non-sedating H1 blockers (e.g. fexofenadine)
c) H2 blocker (e.g. famotidine)
d) Corticosteroid (e.g. prednisone)
e) Antibiotic (e.g. amoxicillin)
47) An infant presents with high fever, lymphadenopathy, and mucocutaneous lesions.
Physical exam reveals bilateral conjunctivitis, dry fissured lips, strawberry tongue,
indurative edema of the feet, and truncal polymorphous rash. Kawasaki disease is
suspected. What is the treatment of choice for the convalescent phase?
a) Acetaminophen
b) Aspirin
c) Indomethacin
d) Ibuprofen
e) Hydrocodone
48) Fever of unknown origin (FUO) implies a fever greater than 38.3C degrees (101F)
for equal to or greater than how many days?
a) 3 days

DO NOT DISTRIBUTE - 16 -
Pediatrics 01May2009

b) 5 days
c) 7 days
d) 14 days
e) 21 days
49) The most common bacteria implicated in acute otitis media (AOM) in pediatrics are
S. pneumoniae, H. influenzae, and M. catarrhalis. What is the treatment of choice?
a) Amoxicillin
b) Doxycycline
c) Penicillin
d) Ketoconazole
e) Ceftriaxone
50) In general, children with pharyngitis should not be treated with antibiotics
empirically as most episodes are viral. If Group A Streptococcus (GAS) is suspected,
what is the drug of choice?
a) Amoxicillin
b) Doxycycline
c) Penicillin
d) Ketoconazole
e) Ceftriaxone
51) A 14-year-old boy presents with extreme fatigue. Testing shows fever, generalized
lymphadenopathy, atypical lymphocytes, and a positive heterophile antibody test.
Epstein-Barr virus is suspected. Which of the following is most likely?
a) Croup
b) Epiglottitis
c) Mononucleosis
d) Bronchiolitis
e) Pertussis
52) A child arrives at the Emergency Department with a hoarse voice and barky seal-like
cough. Physical exam reveals stridor and the clinician orders steroids and nebulized
epinephrine. Parainfluenza virus is suspected. Which of the following is most likely?
a) Croup
b) Epiglottitis
c) Mononucleosis
d) Bronchiolitis
e) Pertussis
53) What is the treatment of choice for epiglottitis?
a) Supportive only
b) IV epinephrine
c) IV prednisone
d) IV amoxicillin
e) Endotracheal intubation
54) Bronchiolitis is characterized by 5-10 days of wheezing, rhonchi, and crackles
typically between November and April. Prophylactic use of palivizumab may be
beneficial. What is the most common cause of bronchiolitis in children?
a) Influenza virus
b) Parainfluenza virus

DO NOT DISTRIBUTE - 17 -
Pediatrics 01May2009

c) Respiratory syncytial virus


d) Epstein-Barr virus
e) S. pneumoniae
55) A child presents to the Emergency Department with a long, stridorous inspiration
after a paroxysmal “whooping” cough. The child is started on erythromycin estolate, a
macrolide. Which of the following is most likely?
a) Croup
b) Epiglottitis
c) Mononucleosis
d) Bronchiolitis
e) Pertussis
56) A patient is asked to come into her obstetrician at 35-weeks of gestation to get a
bacteria test. The test will help determine if she will receive penicillin during labor to
prevent neonatal pneumonia and meningitis. Which of the following is being tested for?
a) Streptococcus pneumoniae
b) Haemophilus influenzae
c) Mycoplasma pneumoniae
d) Group B strep (S. agalactiae)
e) Group D strep (Enterococcus)
57) An adolescent child (school age) presents with photophobia, fever, and neck pain.
Testing reveals a positive Brudzinski sign and a positive Kernig test. Which of the
following is most likely?
a) Streptococcus pneumoniae
b) Haemophilus influenzae
c) Mycoplasma pneumoniae
d) Group B strep (S. agalactiae)
e) Group D strep (Enterococcus)
58) Bacterial meningitis involving S. pneumoniae or H. influenzae is most commonly
seen in what population?
a) Neonates
b) Children ages 3-6
c) Children ages 6-14
d) Teenagers
e) Adults
59) What is the most common cause of gastroenteritis in infants and toddlers?
a) Giardia lamblia
b) Campylobacter jejuni
c) Yersinia enterocolitica
d) Norwalk virus
e) Rotavirus
Match the form of gastroenteritis with the treatment:
60) Shigellosis a) No antibiotics unless systemic
61) C. jejuni b) Stop antibiotics, metronidazole
62) Salmonella c) TMP/SMX
63) C. difficile d) Erythromycin
64) Giardiasis e) Metronidazole

DO NOT DISTRIBUTE - 18 -
Pediatrics 01May2009

65) A child from the South Atlantic U.S. presents with a headache and rash that began
about a week after a hike in the woods. The rash began on the wrists and ankles and
spread proximally to the trunk. What is the drug of choice?
a) Amoxicillin
b) Ceftriaxone
c) Penicillin
d) Doxycycline
e) Clindamycin
66) A five-year-old child is brought in with multiple erythema migrans lesions. Physical
exam reveals lymphadenopathy and cranial nerve palsy. ECG shows a slight AV block.
History reveals the child’s brothers brought him into the Minnesota woods to go hunting,
but the child was never examined for ticks after the trip. What is the initial drug of choice
for this child, who is younger than 8-years-old?
a) Amoxicillin
b) TMP/SMX
c) Ketoconazole
d) Doxycycline
e) Clindamycin

Pediatrics #3 – Clinical: Part Three


1) What is the Apgar (APGAR) score for a child with a heart rate of 90, irregular and
weak cry, cyanotic extremities, weak and slightly flexed extremities, and grimacing facial
expression?
a) 0
b) 3
c) 5
d) 7
e) 9
2) Which of the following forms of birth trauma involves a diffuse, edematous, and often
dark swelling of the soft tissues of the scalp that extends across the midline and/or suture
lines?
a) Erb palsy
b) Fractured clavicle
c) Klumpke palsy
d) Caput succedaneum
e) Cephalhematoma
3) A newborn presents with an arm that is extended, internally rotated, and flexed at the
wrist. Moro reflex of the right arm is absent but right hand grasp is intact. Which of the
following is most likely?
a) Erb palsy
b) Fractured clavicle
c) Klumpke palsy
d) Caput succedaneum
e) Cephalhematoma
4) Very low birth weight (VLBW) is defined as less than:
a) 1,500g

DO NOT DISTRIBUTE - 19 -
Pediatrics 01May2009

b) 2,500g
c) 3,500g
d) 4,500g
e) 5,500g
5) Which of the following is NOT common in postmature (>42 weeks) infants?
a) Meconium aspiration
b) Persistent pulmonary hypertension
c) Hyperglycemia
d) Hypocalcemia
e) Polycythemia
6) Which of the following congenital infections presents with cataracts, “blueberry
muffin” skin syndrome, vertical bone striation, and patent ductus?
a) Toxoplasma gondii
b) Treponema pallidum
c) Rubella
d) Cytomegalovirus
e) Herpes simplex
7) Which of the following congenital infections is the most common and presents with
skin vesicles or denuded skin, keratoconjunctivitis, and seizures?
a) Toxoplasma gondii
b) Treponema pallidum
c) Rubella
d) Cytomegalovirus
e) Herpes simplex
8) Early-onset neonatal sepsis usually involves bacteria that colonize the mother’s
genitourinary tract. On the other hand, nosocomial-acquired sepsis involves drug-
resistant bacterial pathogens that are more commonly seen in the neonatal intensive care
unit (NICU), such as:
a) Group B strep (agalactiae)
b) E. coli
c) Klebsiella
d) S. aureus
e) L. monocytogenes
9) Respiratory distress syndrome (RDS) involves the formation of a hyaline membrane
(hazy ground-glass appearance chest radiograph) and a deficiency in surfactant.
Measuring the lecithin-to-sphingomyelin ratio can help predict RDS. What is the major
factor predisposing a newborn to RDS?
a) Birth involving meconium
b) Breech presentation
c) Delayed cord clamping
d) Pre-maturity (<34 weeks)
e) Post-maturity (>45 weeks)
10) What is the most common cause of neonatal unconjugated hyperbilirubinemia?
a) Post-birth UV-light exposure
b) Hemolytic disorder
c) Bacterial sepsis

DO NOT DISTRIBUTE - 20 -
Pediatrics 01May2009

d) Extrahepatic obstruction
e) Genetic disorder (e.g. Rotor, Dubin-Johnson)
11) Necrotizing enterocolitis (NEC) is seen in premature infants. Which of the following
is the most common cause of neonatal polycythemia?
a) Birth involving meconium
b) Breech presentation
c) Delayed cord clamping
d) Pre-maturity (<34 weeks)
e) Post-maturity (>45 weeks)
12) A newborn has a positive Coombs test, spherocytes, increased bilirubin, and
increased reticulocytes. Which of the following is most likely?
a) Anemia of infancy
b) Intraventricular hemorrhage
c) Immune hemolysis
d) Hereditary spherocytosis
e) Glucose-6-phosphate dehydrogenase deficiency
13) A preterm infant is found to have bleeding of the germinal matrix, an area of
immature vasculature that is the site of pluripotent cells that migrate to form neurons and
glia. Which of the following is most likely?
a) Anemia of infancy
b) Intraventricular hemorrhage
c) Immune hemolysis
d) Hereditary spherocytosis
e) Glucose-6-phosphate dehydrogenase deficiency
14) What is the drug of choice for neonatal seizures?
a) Phenobarbital
b) Succinylcholine
c) Midazolam
d) Fentanyl
e) Rohypnol
15) All states require newborn screening for hypothyroidism. Which of the following
would be suggestive of primary hypothyroidism?
a) Low T4 level and low TSH level
b) Low T4 level and high TSH level
c) High T4 level and low TSH level
d) High T4 level and high TSH level
16) Which of the following describes infants with Beckwith-Wiedemann syndrome or
islet cell adenomas?
a) Transient hypoglycemia
b) Protracted hypoglycemia
c) Transient hyperinsulinemia
d) Protracted hyperinsulinemia
17) What is the most common type of tracheoesophageal fistula (TEF)?
a) Distal TEF with esophageal atresia
b) H-type TEF without esophageal atresia
c) Proximal TEF with esophageal atresia

DO NOT DISTRIBUTE - 21 -
Pediatrics 01May2009

d) Proximal and distal TEF with esophageal atresia


e) Esophageal atresia without TEF (no air in GI tract)
18) A double bubble sign on radiograph is indicative of:
a) Distal TEF
b) Proximal TEF
c) Gastric atresia
d) Duodenal atresia
e) Bile duct atresia
19) What defect is seen here?
a) Gastroschisis
b) Meckel diverticulum
c) Hirschsprung disease
d) Omphalocele
e) Diaphragmatic hernia
20) Failure of the tongue to descent by the 9th week of gestation causes:
a) Unilateral cleft lip
b) Bilateral cleft lip
c) Midline cleft palate
d) Lateral cleft palate
21) Which of the following are transient dark blue-black pigmented macules over the
lower back and buttocks that are sharply demarked (do not fade into surrounding skin)?
a) Child abuse bruises
b) Erythema toxicum neonatorum
c) Seborrheic dermatitis
d) Mongolian spots
22) A newborn with IUGR is born with microcephaly and mental retardation. Physical
exam reveals mid-facial hypoplasia, micrognathia, and a flattened philtrum. Which of the
following did the mother likely use during pregnancy?
a) Cocaine
b) Heroine
c) Methamphetamine
d) Marijuana
e) Alcohol
23) Infants of narcotic-abusing mothers should never be given naloxone (Narcan) in the
delivery room as it can precipitate:
a) Seizures
b) Myocardial infarction
c) Stroke
d) Respiratory distress syndrome
e) Ascites
24) Multicystic kidney is always unilateral and is the most common type of renal
dysplasia. It consists of numerous non-communicating, fluid filled cysts, and a diagnosis
is best confirmed by:
a) MRI
b) CT scan
c) Abdominal x-ray

DO NOT DISTRIBUTE - 22 -
Pediatrics 01May2009

d) Ultrasound
e) Barium study
25) What is the most common cause of hydronephrosis in childhood?
a) Ureteropelvic junction obstruction
b) Ureterovesical junction obstruction
c) Autosomal dominate polycystic kidney disease
d) Autosomal recessive polycystic kidney disease
e) Posterior urethral valves
26) What is the treatment of choice for infants with recurrent UTIs caused by
vesicoureteral reflex (VUR)?
a) Nitrofurantoin
b) TMP/SMX
c) Amoxicillin
d) Ciprofloxacin
e) Ceftriaxone
27) What is the most common cause of end-stage renal disease in childhood?
a) Ureteropelvic junction obstruction
b) Ureterovesical junction obstruction
c) Autosomal dominate polycystic kidney disease
d) Autosomal recessive polycystic kidney disease
e) Posterior urethral valves
28) During an exam of an adolescent boy, the left side of the scrotal sac appears like a
bag of worms. Palpation reveals tortuous veins. Which of the following is most likely?
a) Varicocele
b) Hydrocele
c) Testicular torsion
d) Hypospadias
e) Cryptorchidism
29) What is the most common pathogen seen in pediatric UTIs?
a) S. aureus
b) C. difficile
c) E. coli
d) H. influenzae
e) S. saprophyticus
30) A child is found to have extreme proteinuria, hypoalbuminemia, hyperlipidemia, and
edema. What is the most common cause of primary nephrotic syndrome in the pediatric
population?
a) Minimal change disease
b) Cystic renal dysplasia
c) Membranous glomerulopathy
d) Acute post-streptococcal glomerulonephritis
e) Membranoproliferative glomerulonephritis
31) A child presents with hematuria that is overt on microscopic examination and
contains red cell casts. What is the most common glomerulonephritis seen in childhood?
a) Minimal change disease
b) Cystic renal dysplasia

DO NOT DISTRIBUTE - 23 -
Pediatrics 01May2009

c) Membranous glomerulopathy
d) Acute post-streptococcal glomerulonephritis
e) Membranoproliferative glomerulonephritis
32) A child presents with painless hematuria and sensorineural hearing loss. Damage to
type IV collagen is suspected (Alport syndrome). How is this disorder inherited?
a) Autosomal dominant
b) Autosomal recessive
c) X-linked
d) Y-linked
e) Mitochondrial
33) Which of the following is the most common type of renal tubular acidosis (RTA)
seen in children and adults?
a) Distal Type I
b) Distal Type IV
c) Proximal Type II with Fanconi syndrome
d) Proximal Type II without Fanconi syndrome
34) A patient is found to have a low serum pH and low serum bicarbonate. A workup for
hyperchloremic metabolic acidosis is begun. The patient’s urine anion gap (Na+ + K+ -
Cl-) is found to be positive. The patient is hyperkalemic and has a urine pH < 5.5. Which
of the following describes this patient?
a) Normal
b) Proximal RTA (type 2)
c) Distal RTA (type 1)
d) Distal RTA (type 4)
e) None of the above
35) Which of the following would cause diabetes insipidus (DI)?
a) Excess ADH release
b) Inability to release ADH
c) Excess ACTH release
d) Inability to release ACTH
e) Hyperglycemia
36) What is the most common form of acute renal failure (ARF) in the pediatric
population?
a) Pre-renal
b) Intra-renal (intrinsic)
c) Post-renal
37) What is the prophylaxis for preventing neural tube defects, which may be diagnosed
by an increase in maternal alpha-fetoprotein?
a) Niacin
b) Diabetes control
c) Electrolyte balance
d) Sodium control
e) Folic acid
38) A newborn is found to have a tuft of hair on the lower back. Testing shows an
underlying bony vertebral lesion without herniation of any spinal contents. Which of the
following is most likely?

DO NOT DISTRIBUTE - 24 -
Pediatrics 01May2009

a) Myelomeningocele
b) Meningocele
c) Spina bifida occulta
d) Arnold-Chari malformation
39) A child is found to have an Arnold-Chari malformation and begins developing signs
of hydrocephalus. Which of the following is NOT a component of the Cushing triad?
a) Upward gaze paralysis (“setting sun” sign)
b) Bradycardia
c) Hypertension
d) Cheyne-Stokes respiration
40) Which of the following is the term for bilateral lower extremity spasticity, which may
be seen in cerebral palsy?
a) Diplegia
b) Hemiplegia
c) Quadriplegia
d) Seizure
41) A child is bought in for neurologic screening after the mother finds him “blanking
out” for a few seconds. She says the boy will be talking, stop taking, then pick back up at
the same part of the sentence about 5 seconds later. EEG during one of these episodes
shows a characteristic 3-per-second spike and wave pattern. Which of the following is
most likely?
a) Generalized seizure
b) Petit mal seizure
c) Infantile spasm
d) Febrile seizure
42) Reye syndrome is characterized by acute-onset encephalopathy and degenerative
liver disease when what drug is given following a viral illness (e.g. chicken pox)?
a) Acetaminophen
b) Aspirin
c) Ibuprofen
d) Naproxen
e) Indomethacin
43) A child presents with complaints of weakness. History reveals a viral illness about a
week earlier. The weakness has been progressive, ascending, and of acute onset. Testing
reveals likely autoimmune-mediated demyelination of peripheral nerves. Plasmapheresis
is started to hasten resolution. Which of the following is most likely?
a) Duchenne muscular dystrophy (DMD)
b) Becker muscular dystrophy
c) Guillain-Barré syndrome
d) Myasthenia gravis (MG)
e) Multiple sclerosis (MS)
44) Which of the following is an autoimmune disorder of the neuromuscular junction
with autoantibodies binding to postsynaptic Ach receptors?
a) Duchenne muscular dystrophy (DMD)
b) Becker muscular dystrophy
c) Guillain-Barré syndrome

DO NOT DISTRIBUTE - 25 -
Pediatrics 01May2009

d) Myasthenia gravis (MG)


e) Multiple sclerosis (MS)
45) A child presents with motor difficulties. Physical exam reveals hypertrophy of the
calves. Watching the child stand up involves them pushing their trunk up with their arms,
then standing (Gower sign). An X-linked recessive disorder is suspected. Which of the
following is most likely?
a) Duchenne muscular dystrophy (DMD)
b) Multiple sclerosis (MS)
c) Guillain-Barré syndrome
d) Myasthenia gravis (MG)
e) Spinal muscle atrophy (SMA)
46) Which of the following primarily affects the anterior horn cell of the spine and has
mostly been eradicated due to a killed virus vaccination?
a) Duchenne muscular dystrophy (DMD)
b) SMA Type I (Werdnig-Hoffman disease)
c) SMA Type II
d) Poliomyelitis
e) Guillain-Barré syndrome
47) Hydrocephalus is most associated with:
a) Head size two standard deviations below mean (microcephaly)
b) Head size one standard deviation below mean
c) Abnormally shaped head (craniosynostosis)
d) Head size one standard deviation above mean
e) Head size two standard deviations above mean (macrocephaly)
48) Failure to thrive (FTT) is defined as weight below what percentile?
a) 10th
b) 7th
c) 5th
d) 3rd
e) 1st
49) Pediatric obesity is defined as weight above what percentile?
a) 99th
b) 97th
c) 95th
d) 93rd
e) 90th
50) What is the most common cause of amblyopia in children?
a) Retinal trauma
b) Strabismus
c) Retinoblastoma
d) Congenital cataract
e) Retinopathy of prematurity (ROP)
51) What is the most common intraocular malignancy of childhood, which presents with
leukocoria?
a) Malignant melanoma
b) Neuroblastoma

DO NOT DISTRIBUTE - 26 -
Pediatrics 01May2009

c) Retinoblastoma
d) Metastatic adenocarcinoma
e) Ocular carcinoma
52) What is the most common cause of leukocoria in children?
a) Retinal trauma
b) Strabismus
c) Retinoblastoma
d) Congenital cataract
e) Retinopathy of prematurity (ROP)
53) Fluorescein drops with a blue-filter light would most likely be used in which of the
following cases?
a) Corneal abrasions
b) Retinoblastoma
c) Congenital cataracts
d) Retinopathy of prematurity
e) Malignant melanoma
54) Steroid drops should NOT be given in which of the following causes of
conjunctivitis?
a) Neisseria gonorrhoeae
b) Corynebacterium
c) Streptococci
d) Staphylococci
e) HSV1
55) An infant presents with suspected developmental dysplasia of the hip (DHH). A
Barlow maneuver is positive and an Ortolani maneuver confirms the finding as the hip
being abnormally positioned through most of the exam. Which is most likely?
a) Subluxatable
b) Dislocatable
c) Dislocated
d) Fused
56) Limp is the most common musculoskeletal complaint in children. What is the most
common cause of limp?
a) Talipes equinovarus (clubfoot)
b) Metatarsus adductus
c) Genu valgum
d) Genu varum
e) Trauma
57) Legg-Calvé-Perthes disease is defined as avascular necrosis of the:
a) Navicular bone of the hand
b) Lunate bone of the hand
c) Sacroiliac joint
d) Femoral head
e) Talus bone of the ankle
58) Which of the following is NOT a contributing factor to slipped capital femoris
epiphysis (SCFE)?
a) Trauma

DO NOT DISTRIBUTE - 27 -
Pediatrics 01May2009

b) Weight
c) Male gender
d) Puberty
59) A young male child presents with painless limp and knee pain. Which of the
following is most likely?
a) SCFE
b) Duchenne muscular dystrophy
c) Legg-Calvé-Perthes disease
d) DHH
e) PCL tear
60) Which of the following is particularly common in toddlers due to twisting forces on
the tibia during falling?
a) Torus (buckle) fracture
b) Greenstick fracture
c) Spiral fracture
d) Salter-Harris fracture type V
e) Salter-Harris fracture type IV
61) What type of osteogenesis imperfecta (OI) is autosomal dominant, has conductive
hearing loss, blue sclerae, and presents with bow legs or neonatal fractures?
a) Type I
b) Type II
c) Type III
d) Type IV
62) A child is brought to the clinic by his mother after the child’s father was swinging the
child around the living room by his arms. The child holds his right arm close to the body,
slightly flexing and pronating the hand. Motion at the elbow is limited. Which of the
following is most likely?
a) Osteomyelitis
b) Osteogenesis imperfecta
c) Lateral epicondylitis
d) Medial epicondylitis
e) Radial head subluxation
63) What is the most common pathogen seen in osteomyelitis in children?
a) Shigella
b) Salmonella
c) S. aureus
d) S. epidermidis
e) N. gonorrhoeae
64) An adolescent presents with a painful knee, fever, and refusal to bear weight on the
affected limb. Aspiration reveals diplococci that grow on chocolate agar with carbon
dioxide, ruling out the most common cause. Which of the following is most likely?
a) Shigella
b) Salmonella
c) S. aureus
d) S. epidermidis
e) N. gonorrhoeae

DO NOT DISTRIBUTE - 28 -
Pediatrics 01May2009

65) What is the most common reason for hospitalization in pediatric practice?
a) Renal dysfunction
b) Congenital heart defects
c) Asthma
d) Fever of unknown origin
66) A child presents with acute respiratory distress with dyspnea and wheezing. Exam
reveals subcostal retractions, tripod positioning, tracheal tugging, and nasal flaring.
Which of the following drugs would be most useful for this acute attack?
a) Beclomethasone (inhaled corticosteroid)
b) Flunisolide (inhaled corticosteroid)
c) Epinephrine IV
d) Albuterol (beta2-agonist)
e) Montelukast (leukotriene receptor antagonist)
67) A newborn presents with failure to thrive and pulmonary problems. An abdominal
radiograph shows a mottle appearance, suggesting meconium ileus. Elevated levels of
what ion would be expected in this child’s sweat?
a) Potassium
b) Sodium
c) Calcium
d) Magnesium
e) Chloride
68) A mother brings in her 2-month-old son to the pediatrician. The mother states the
child rolled off the sofa onto the carpeted floor. Testing shows subdural hematomas.
Physical exam reveals retinal hemorrhages. Which of the following is most likely?
a) Caput succedaneum
b) Cephalhematoma
c) Traumatic fall
d) Child abuse

Pediatrics #4 – Extra: New Treatments For Multiple Sclerosis


1) Which of the following is NOT true of multiple sclerosis (MS)?
a) Affects more than 30 per 100,000 in middle North America
b) Has a higher prevalence in temperate zones
c) Most commonly affects women between ages 20 and 40
d) In most cases, is caused by a viral infection
e) Genetic factor may play a role in susceptibility
2) Which of the following is characteristic of phase-2 MS, not phase-1?
a) Hyperreflexic spasticity
b) Bladder and bowel affected
c) Hyperreflexia with good axial tone
d) Diminution of deep tendon reflexes
e) Ataxia of gait with asymmetric paraparesis
3) Which of the following is characteristic of phase-2 MS, not phase-3?
a) Bulbar difficulties with swallowing
b) Gaze palsies
c) Frequency of urination and inability to completely void

DO NOT DISTRIBUTE - 29 -
Pediatrics 01May2009

d) Bulbar difficulties with speaking


e) Limb and trunk spasticity
4) Which of the following during early-onset MS correlates to a more severe course of
disease?
a) Length of initial event
b) Loss of urinary control
c) Number of relapses
d) Intensity of initial event
e) The appears to be no correlation between early-onset and disease course
5) A young adult with MS has ocular pain, abnormal visual acuity and fields, reduced
vision, relative afferent pupillary defect, and abnormal visual evoked potentials (VEPs).
Which of the following is most likely?
a) Uveitis
b) Glaucoma
c) Retinoblastoma
d) Coats disease
e) Optic neuritis
6) What is the central pathological CNS event suspected in the etiology of MS?
a) Destruction of myelin
b) Destruction of the putamen
c) Destruction of the caudate nucleus
d) Destruction of the substantia nigra
e) Lysosomal accumulation of lipids
7) The T2-lesion volume (gadolinium-enhanced) on MRI at what locations may be an
associated predictor of depression, which occurs in 25-50% of patients with MS?
a) Left anterior inferior prefrontal cortex and left anterior temporal CSF
b) Right anterior inferior prefrontal cortex and right anterior temporal CSF
c) Left anterior inferior prefrontal cortex and right anterior temporal CSF
d) Right anterior inferior prefrontal cortex and left anterior temporal CSF
8) Which of the following tests is useful in evaluating demyelination in the posterior
columns of the spinal cord?
a) Visual evoked response (VER) test
b) Sensory evoked response (SER) test
c) Rapid plasma reagin (RPR) test
d) Erythrocyte sedimentation rate (ESR)
e) Vitamin B12 levels
9) The presence of gamma globulins with oligoclonal bands within the CSF indicates:
a) The destruction of myelin
b) The extent of future MS disease course
c) The synthesis of immunoglobulins in the CNS
d) The destruction of autoantibodies
e) An autoimmune reaction in the PNS
10) What is the treatment for acute exacerbations of multiple sclerosis?
a) Prednisone 500mg IV BID for 3-5 days
b) Prednisone 100mg IV QID for 3-5 days
c) Prednisone 1000mg IM QD for 3-5 days

DO NOT DISTRIBUTE - 30 -
Pediatrics 01May2009

d) Methylprednisolone 100mg IV BID for 3-5 days


e) Methylprednisolone 1000mg IV QD for 3-5 days
11) Which of the following has been reported to increase in incidence in MS patient
receiving interferon-beta?
a) Headaches
b) Influenza-like symptoms
c) Asthenia
d) Anemia
e) Seizures
12) Modafinil and amantadine are used in MS for patients complaining of:
a) Spasticity
b) Depression
c) Insomnia
d) Fatigue
e) Nausea
13) 3,4-Diaminopyridine (3,4-DAP) may improve motor function and fatigue in some
patients with MS. What is its mechanism of action?
a) Sodium channel blocker
b) Beta blocker
c) Potassium channel blocker
d) Calcium channel blocker
e) Alpha adrenergic agonist
14) Baclofen and tizanidine are used in MS for patients complaining of:
a) Spasticity
b) Depression
c) Insomnia
d) Fatigue
e) Nausea
15) Tolterodine may be used for hyperactive bladder in MS patients. Which of the
following is a common side effect of this drug?
a) Vomiting
b) Dry mouth
c) Bradycardia
d) Conjunctival injection
e) Profuse sweating
16) Which of the following is NOT part of the Barthof criteria in the diagnosis of MS,
which requires 3 of 6 criteria to be met?
a) Objective neurologic abnormalities on clinical examination with involvement
of white matter long tracts
b) One area of CNS involvement
c) Two or more episodes of progression over 6 months
d) Age between 15 and 60 years
e) Exclusion of other diseases that may produce similar symptoms
f) Presence of oligoclonal bands and/or myelin basic protein in CSF
17) Which of the following disease-modifying drugs for MS has the highest cost and is
dosed as 20mg SC once a day?

DO NOT DISTRIBUTE - 31 -
Pediatrics 01May2009

a) Interferon beta-1a
b) Interferon beta-1b
c) Glatiramer
d) Mitoxantrone
e) Natalizumab
18) Which of the following has been shown to decrease the number and volume of
gadolinium-enhancing lesions in MS patients, with a decline of about 44%?
a) Botulinum-A toxin (Botox)
b) Amantadine (Symmetrel)
c) Simvastatin (Zocor)
d) Natalizumab (Tysabri)
e) Tolterodine (Detrol)
19) Which of the following time periods shows an increase in MS relapses?
a) Pre-pregnancy
b) First trimester
c) Third trimester
d) First 3-months post pregnancy
e) 3 to 6-months post pregnancy

Pediatrics #5 – Clinical: Textbook Questions


1) A 2-year-old female child presents with VT, severe ventricular dysfunction,
hypotension, and metabolic acidosis. The patient is cardioverted into ventricular
fibrillation, which degenerates into asystole. What is the most appropriate indication for
using intravenous epinephrine in this patient?
a) Ventricular ectopy
b) Asystole
c) Severe refractory metabolic acidosis and/or hyperkalemia
d) Bradycardia
e) Supraventricular tachycardia
2) A 16-year-old female patient presents with short stature and no secondary sexual
characteristics. What diagnosis must be considered?
a) Turner syndrome
b) Isolated growth hormone deficiency
c) Cushing disease
d) Familial short stature
e) Addison disease
3) Galactosemia, a disorder of carbohydrate metabolism, is inherited in an autosomal
recessive fashion. What is the risk of galactosemia in a child whose parents are both
carriers for the disorder?
a) 100%
b) 75%
c) 50%
d) 25%
e) 0%
4) Which of the following statements is true regarding children with sickle cell disease?
a) Vaccinations are not required because they receive penicillin prophylaxis

DO NOT DISTRIBUTE - 32 -
Pediatrics 01May2009

b) Gallstones typically develop before the age of 3 years


c) Episodes of dactylitis should be treated with antibiotics
d) Hydroxyurea maintenance therapy decreases the number and severity of vaso-
occlusive crises
e) Acute chest syndrome requires only supportive care
5) A mother brings her 5-year-old son to your office in New Mexico for his regular health
maintenance visit. A quick review of the patient's chart reveals that he and his family are
strict vegans. Their house is very small, so all the children spend a good deal of time
outside. The mother states that her son eats plenty of dark green vegetables and iron-
fortified grains. She does not believe in providing supplemental vitamins and minerals.
This child is most at risk for nutritional deficiency involving which of the following?
a) Vitamin B12
b) Vitamin B6
c) Niacin
d) Riboflavin
e) Vitamin D
6) A 6-year-old boy presents with a newly appreciated heart murmur. He is
asymptomatic, with normal growth and development and normal exercise tolerance. On
examination S1 and S2 are normal; a II/VI low-frequency midsystolic murmur is heard at
the left lower sternal border. His pulses are normal. The most likely diagnosis is:
a) Bicuspid aortic valve
b) Still's murmur
c) Ventricular septal defect
d) Atrial septal defect
e) Coarctation of the aorta
7) You are called to the delivery room for a routine birth. The infant cries when the cord
is cut. You examine the child under the warmer and notice that when he stops crying, his
chest heaves, and he turns blue. You are unable to pass the NG tube through the nose for
suctioning. Which condition is most likely causing this infant's respiratory distress?
a) Choanal atresia or stenosis
b) Vocal cord paralysis
c) Subglottic stenosis
d) Recurrent laryngeal nerve damage
e) Laryngeal web
8) A 3-year-old girl is diagnosed with new-onset insulin-dependent diabetes mellitus.
Which of the following laboratory findings is consistent with diabetic ketoacidosis?
a) Hypoglycemia
b) Hypercarbia
c) Ketones in urine
d) Increased venous blood pH
e) Decreased BUN
9) During a male newborn examination, the testes are not palpable in the scrotal sacs.
One testis is palpable high in the right inguinal canal and cannot be gently manipulated
into the anatomically correct position. The left testis is not palpable but is discovered in
the abdomen after consultation with a pediatric urologist and an abdominal ultrasound. In
counseling the parents, which one of these statements regarding cryptorchidism is true?

DO NOT DISTRIBUTE - 33 -
Pediatrics 01May2009

a) More than 99% of males have bilateral descended testes at age 1 year
b) Impaired sperm production is not a concern if neither testis descends
c) Malignant degeneration is not a risk factor for testes, which do not descend as
long as they are placed within the scrotal sac through surgery by 1 year of age
d) This infant is no more likely than his peers to manifest an inguinal hernia
e) Microphallus is a common associated condition
10) A 5-year-old boy presents with a waddling limp and has had a stiff right hip for the
last 2 months. He has minimal complaints of pain. The most likely diagnosis is:
a) Legg-Calve-Perthes disease
b) Slipped capital femoral epiphysis
c) Toddler's fracture
d) Septic arthritis
e) Juvenile idiopathic arthritis
11) A 17-year-old young girl on oral contraceptive therapy for regulation of her
menstrual periods presents with a 1-week history of left leg pain and swelling. Evaluation
with a Doppler ultrasound reveals absence of flow in the left femoral and popliteal veins.
The clot extends proximally to the left external iliac vein. The most important potential
complication that one should be cautious about in this girl is:
a) Venous insufficiency
b) Limb overgrowth
c) Pulmonary embolism
d) Edema
e) Gangrene
12) A woman with a seizure disorder under medical management wants to conceive a
child. Her risk of having a child with a neural tube defect is greatest if her current
medical regimen includes which of the following?
a) Phenobarbital
b) Phenytoin
c) Ethosuximide
d) Carbamazepine
e) Primidone
13) A 2-month-old infant presents to your emergency department with a heart rate of 220
beats/minute, pulses, and adequate perfusion. After giving the infant oxygen, you note
abnormal P waves and a narrow QRS (!0.08 sec) on the cardiac monitor. Which of the
following is the best course of action?
a) Administer IV/IO epinephrine
b) Administer IV adenosine by rapid bolus
c) Administer IV calcium chloride
d) Administer IV atropine by rapid bolus
e) Administer IV sodium bicarbonate
14) A 3-month-old infant presents with a history of abnormal movements that his parents
think might be seizures. You observe an episode of recurrent rhythmic flexor-extensor
spasms that repeat about 30 times before subsiding. The EEG shows hypsarrhythmia, and
a Wood lamp exam is positive for several flat, hypopigmented macules scattered over the
skin surface. This child's infantile spasms are most likely a result of which of the
following underlying disorders?

DO NOT DISTRIBUTE - 34 -
Pediatrics 01May2009

a) Von Recklinghausen disease


b) Tuberous sclerosis
c) Von Hippel-Lindau disease
d) Sturge-Weber disease
e) Bilateral acoustic neurofibromatosis
15) A 21-month-old girl arrives at clinic in May with a vaccination record that indicates
that she has received 3 DTaP doses, 3 Hib doses, 3 IPV doses, 3 pneumococcal conjugate
vaccine doses, 2 hepatitis A vaccine doses, and 3 hepatitis B vaccine doses. Which of the
following should be administered at this visit?
a) DTaP, Hib, IPV, varicella
b) DTaP, Hib, pneumococcal conjugate vaccine, MMR, and varicella
c) DTaP, hepatitis A, IPV, pneumococcal conjugate vaccine
d) DTaP, hepatitis B, MMR, and varicella
e) DTaP, hepatitis A, IPV, MMR, and varicella
16) The mother of a 30-month-old boy is concerned that the child's speech is “garbled.”
The child uses “ma-ma” and “da-da” appropriately. He uses about 30 other words, but
most of them are mispronounced (for instance, “boo” instead of “blue”). The boy's aunt,
uncle, and cousins came to visit for a weekend and were unable to understand more than
half of what he said. Examination of the ears reveals normal canals with translucent,
mobile tympanic membranes, and visible landmarks. Which of the following evaluations
for speech delay should be performed first?
a) Receptive language testing
b) Phonetic testing
c) Dysfluency evaluation
d) Tympanogram testing
e) Audiologic (hearing) assessment
17) A 13-year-old girl presents with recurrent abdominal pain over the last 3 months. She
has missed a total of 8 days of school. There is no associated fever, weight loss,
gastrointestinal bleeding, and the pain does not occur in relation to meals or awaken her
from sleep. There is diffuse abdominal tenderness but no other abnormal findings on
examination. Which approach is likely to help in the diagnosis and management of her
condition?
a) Abdominal CT scan with contrast
b) Upper and lower endoscopy and biopsies
c) Explaining the likely etiology of her symptoms using a biopsychosocial model
and symptomatic therapy
d) A diet history and a diet elimination trial
e) Referral to a psychiatrist
18) A newborn male child has a flat facial profile, upslanted palpebral fissures, epicanthal
folds, a small mouth with a protruding tongue, small genitalia, and simian creases on his
hands. What of the following chromosomal disorders is most likely in this child?
a) Trisomy 21
b) Trisomy 18
c) Trisomy 13
d) Klinefelter syndrome
e) Turner syndrome

DO NOT DISTRIBUTE - 35 -
Pediatrics 01May2009

19) At a 2-year well-child visit, you collect information that your patient lives in a very
old rental home with peeling paint. Both the capillary (screening) and venous blood lead
measurements are 50 "g/dL. The patient has a history of constipation but is otherwise
asymptomatic. Which of the following courses of action is most appropriate?
a) Initiate chelation therapy in a lead-free environment within 48 hours
b) Redraw the blood lead level in 1 week and test all siblings; treat if #50 "g/dL
c) Optimize calcium and iron intake and repeat the blood lead level in 1 month;
treat if #50 "g/dL
d) Refer the family to a lead-removal company; repeat the blood lead level 1
month after decontamination of the home, and treat if #50 "g/dL
e) Refer the case to child protective services for parental neglect
20) A young couple is in your office for their prenatal visit, and you are discussing infant
feeding. The father states that he prefers that the mother breastfeed the baby. The mother
is hesitant to commit to breastfeeding because she plans on returning to full-time
employment 6 weeks after the child is born. Neither her mother nor her sisters chose to
breastfeed. She is concerned that human breast milk may not provide all the nutrients that
the child needs, and she believes formula is a more complete nutritional source for
infants. She is willing to consider exclusive breastfeeding based on the American
Academy of Pediatrics recommendation. If her baby is exclusively breastfed, when
should the child begin receiving oral vitamin D supplementation?
a) Never
b) Within the first month of life
c) Age 2 months
d) Age 4 months
e) Age 6 months
21) A 12-year-old female patient presents with fever, night sweats, weight loss, fatigue,
anorexia, and painless, rubbery, cervical lymphadenopathy. What is the most common
presentation of Hodgkin disease?
a) Fever, night sweats, and/or weight loss of >10% in the preceding 6 months
b) Mediastinal lymphadenopathy
c) Painless, rubbery, cervical lymphadenopathy
d) Pruritus
e) Extreme fatigue and anorexia
22) Which of the following medication groupings is most appropriate for a patient 12
years old with persistent asthma who has failed to achieve well-controlled asthma while
receiving step 2 treatment?
a) None
b) A daily low-dose inhaled corticosteroid
c) A daily medium-dose inhaled corticosteroid and a long-acting inhaled $-2-
agonist
d) A daily low-dose inhaled corticosteroid and a long-acting $-agonist
e) A daily medium-dose inhaled corticosteroid and nedocromil
23) Crops of papular, vesicular, pustular lesions starting on the trunk and spreading to the
extremities, in addition to small, irregular red spots with central gray or bluish-white
specks that appear on the buccal mucosa, is the classic description of which of the
following infections?

DO NOT DISTRIBUTE - 36 -
Pediatrics 01May2009

a) Measles
b) Erythema infectiosum (fifth disease)
c) Roseola infantum
d) Zoster (shingles)
e) Rubella
f) Hand-foot-mouth disease
g) Chickenpox
24) A 20-month-old boy who was treated with high-dose amoxicillin (90mg/kg per day)
for acute otitis media 3 weeks ago now presents with acute-onset ear pain, a bulging,
erythematous right tympanic membrane, and decreased mobility on pneumatic otoscopy
examination. Which of the following is the most appropriate antibiotic for this child?
a) Azithromycin
b) Amoxicillin-clavulanate
c) Erythromycin
d) Trimethoprim-sulfamethoxazole
e) Dicloxacillin
25) Which of the following is considered a risk factor for neonatal respiratory distress
syndrome?
a) Neonatal sepsis
b) Poorly controlled maternal diabetes
c) Maternal preeclampsia
d) Neural tube defects
e) Trisomy 21
26) A mildly febrile 6-year-old patient presents to your office with dysuria and urinary
frequency and urgency. She has a history of one prior UTI about 8 months ago. You
obtain a dipstick urinalysis and send a urine culture. The dipstick is positive for nitrites
and leukocyte esterase. Which of the following is the most appropriate course of action at
this time?
a) Await culture results and tailor therapy based on bacterial sensitivities
b) Begin empiric amoxicillin
c) Begin empiric amoxicillin and schedule the child for a renal ultrasound within
the next 6 weeks
d) Begin empiric amoxicillin and schedule the child for a renal ultrasound and
voiding cystourethrogram within the next 6 weeks
e) Admit the child to the hospital for IV ampicillin and gentamycin and schedule a
DMSA scan
27) A 3-month-old infant presents with cyanosis and an echocardiogram reveals that the
child has tetralogy of Fallot. What four associated lesions describe tetralogy of Fallot?
a) Ventricular septal defect, over-riding aorta, pulmonary stenosis, right
ventricular hypertrophy
b) Ventricular septal defect, atrial septal defect, pulmonary stenosis, right
ventricular hypertrophy
c) Ventricular septal defect, atrial septal defect, aortic stenosis, right ventricular
hypertrophy
d) Ventricular septal defect, coarctation of the aorta, aortic stenosis, right
ventricular hypertrophy

DO NOT DISTRIBUTE - 37 -
Pediatrics 01May2009

e) Ventricular septal defect, mitral valve prolapse, pulmonary stenosis, left


ventricular hypertrophy
28) A 3-year-old boy with a known diagnosis of factor XI deficiency presents to the
emergency department with uncontrolled bleeding from a lip laceration following a fall.
The most appropriate product that can be used for factor replacement in this child prior to
suturing is:
a) Cryoprecipitate
b) Granulocyte infusions
c) Fresh frozen plasma (FFP)
d) Platelet transfusion
e) DDAVP
29) At the health maintenance visit for a 12-year-old male, you note that he has entered
his pubertal height growth spurt. The patient's mother asks about what changes her son
should be expecting in his body over the next several years. As part of your review, you
mention that the most typical sequence of pubertal events in males is which of the
following?
a) Peak height velocity, pubarche, penile enlargement, testicular enlargement
b) Peak height velocity, testicular enlargement, penile enlargement, pubarche
c) Testicular enlargement, pubarche, penile enlargement, peak height velocity
d) Testicular enlargement, peak height velocity, penile enlargement, pubarche
e) Pubarche, testicular enlargement, peak height velocity, penile enlargement
30) A 4-year-old child with known asthma presents to the emergency department with a
chief complaint of wheezing for the past 8 hours. On examination he is alert and
cooperative, mildly tachypneic, has diffuse loud expiratory wheeze, and has a pulse
oximetry reading of 89% while breathing room air. He has already taken 3 albuterol
aerosols at home in the past hour. He is unchanged after receiving another albuterol
inhalation treatment in the emergency department. Appropriate next management would
include:
a) Supplemental oxygen
b) Albuterol inhalation
c) Ipratropium bromide inhalation
d) Oral corticosteroids
e) All of the above
31) A previously healthy 3-year-old boy presents with a history of fever and diarrhea for
the past 2 days. The fever has not responded to ibuprofen, and his urine output has
decreased today. On examination, he is alert, has a temperature of 101°F, heart rate of
115 beats per minute, blood pressure of 105/60 mm Hg, and mild diffuse abdominal
tenderness. The serum electrolytes are normal, but his BUN is 60 mg/dL and his serum
creatinine is 1.8 mg/dL. The complete blood count is normal. Urinalysis shows 1+
protein, small blood, and occasional hyaline casts. The kidney ultrasound is normal.
Which of the following statements regarding his acute renal failure is most accurate?
a) It is due to hemolytic-uremic syndrome
b) It is due to pyelonephritis
c) It is due to interstitial nephritis
d) It is due to the use of ibuprofen in a dehydrated state
e) It is due to urinary tract obstruction

DO NOT DISTRIBUTE - 38 -
Pediatrics 01May2009

32) A 14-year-old girl presents with several weeks of profound fatigue, intermittent low-
grade fevers, a facial rash, and joint pain. The rash recently worsened markedly after sun
exposure. On physical examination, she has a malar rash extending over the bridge of the
nose, but sparing the nasolabial folds, painless oral ulcers, and painful limitation of
movement in her wrists and finger joints. On laboratory testing, her WBC is 3,500/mm3,
Hgb 9.5 g/dL, platelet count 120,000/mm3. A urinalysis shows 15 to 19 RBC/hpf and an
elevated protein of 100 mg/dL. Which of the following tests will most likely be positive?
a) Antinuclear (ANA) antibody
b) Rheumatoid factor (RF)
c) Anti-double-stranded DNA (dsDNA) antibody
d) Anti-Smith (Sm) antibody
e) Anti-Ro (SS-A) antibody
33) A 3-month-old female infant presents to your emergency department unresponsive
and with fever, tachypnea, bradycardia, and hypotension. What order should you follow
in your initial assessment?
a) Airway, breathing, circulation, disability, exposure
b) Breathing, airway, circulation, disability, exposure
c) Circulation, airway, breathing, exposure, disability
d) Exposure, breathing, airway, circulation, disability
e) Exposure, airway, breathing, circulation, disability
34) A 4-year-old male child presents with abrupt-onset petechiae and ecchymoses. Other
than the skin findings, the child appears well and is hemodynamically stable. No
splenomegaly is noted. A complete blood count reveals a normal white blood cell count,
a normal hematocrit, and a platelet count of 12,000 per mm3. Large platelets are seen on
the peripheral smear. No premature white cell forms are seen on peripheral smear. The
parent reports that the child had a viral illness 2 weeks before presentation. Which of the
following is the most likely diagnosis?
a) Isoimmune thrombocytopenia
b) Leukemia
c) Sepsis
d) Immune thrombocytopenic purpura
e) Hypersplenism
35) A child presents with a reduced number of CD3+ T cells, an increased number of B
lymphocytes that are mildly abnormal in function, has a conotruncal heart lesion,
hypoplastic thymus, and hypocalcemia. Which of the following chromosomal disorders is
most likely in this child?
a) Zellweger syndrome
b) Microdeletion of 22q11.2
c) Trisomy 13
d) Gaucher disease
e) Wilson disease
36) A 4-month-old former 30-week premature infant is seen in late October for well-child
care. His mother is concerned about the transfusions that the infant required during her
course in the neonatal intensive care unit and wishes to restrict her exposure to blood
products. Referral for administration of which of the following would be most
appropriate to limit her risk of severe bronchiolitis?

DO NOT DISTRIBUTE - 39 -
Pediatrics 01May2009

a) Ribavirin
b) Nasal influenza vaccine
c) Injected influenza vaccine
d) IV RespiGam
e) IM palivizumab
37) The mother of a 2-month-old infant brings her daughter to your office during the
summer for her regular health maintenance visit. The child is cared for by her maternal
grandmother 3 days a week while the mother is at work. The infant is exclusively fed a
cow milk-based commercial formula when she is with the mother; the grandmother
believes that the child should also receive juice diluted with water due to the warm
weather. Which of the following represents the most appropriate dietary counseling
regarding this infant's diet?
a) Formula-fed infants at this age require free water supplementation during warm
months to maintain optimal hydration
b) Formula-fed infants at this age require glucose supplementation during the
warm months to maintain optimal caloric intake
c) Formula-fed infants do not require any additional vitamin, mineral, caloric, or
fluid supplement beyond their formula for the first 6 months of life
d) Dilution of this infant's formula with water or juice on the days that she is with
the maternal grandmother is unnecessary but harmless
e) This infant should be switched to a soy protein-based formula
38) No red reflex is seen on fundoscopic examination of a newborn. Which of the
following is the most likely diagnosis?
a) Retinoblastoma
b) Leukocoria
c) Congenital cataract
d) Congenital glaucoma
e) Toxocariasis
39) A 5-year-old boy is brought to your office complaining of progressive fatigue,
weakness, and nausea over the past few months. He was a model student, but he is now
having trouble in school and displaying frequent outbursts, the last of which resulted in
his being sent home for hitting another child. Initial lab results show mild hypoglycemia,
hyponatremia, and hyperkalemia. The child is diagnosed with adrenal insufficiency and
treated appropriately; however, his behavior continues to worsen, and he begins to have
difficulty walking and speaking. Which of the following is the most likely etiology of his
behavior problems?
a) Tay-Sachs disease
b) Gaucher disease
c) Niemann-Pick disease
d) Adrenoleukodystrophy
e) Rett syndrome
40) An 8-year-old girl thought to have attention-deficit disorder (inattentive-type)
undergoes EEG testing and is found to have a 3-Hz spike-and-wave pattern. Results of
the EEG, coupled with videotaping of episodes of the patient's “inattention,” lead to a
diagnosis of childhood absence epilepsy. Which of the following is most appropriate for
initial treatment of the child's disorder?

DO NOT DISTRIBUTE - 40 -
Pediatrics 01May2009

a) Methylphenidate
b) Carbamazepine
c) ACTH
d) Ethosuximide
e) Phenobarbital
41) A child presents with lymphedema of the hands and feet, a shield-shaped chest,
widely spaced hypoplastic nipples, short stature, and multiple pigmented nevi. In
addition, she had a coarctation of the aorta that was repaired and has renal disease. Her
parents continue to be worried that there is something in addition to her heart condition
that is causing failure to thrive. Which of the following chromosomal disorders is most
likely in this child?
a) Trisomy 21
b) Trisomy 18
c) Trisomy 13
d) Klinefelter syndrome
e) Turner syndrome
42) A 14-year-old patient familiar to the emergency room staff due to multiple visits in
the last 3 months is brought in by her mother for ingestion of an unknown number of
acetaminophen tablets. The mother states that she keeps all the medicines in the house
locked up because “this is just the sort of thing my daughter would do to me.” She saw
the girl stuffing something into her bedside drawer while she was passing the girl's room
and discovered a bottle marked “acetaminophen, 250 tablets.” Only 4 tablets remained in
the bottle. The mother did not believe that the daughter took the tablets until she began
vomiting about an hour later. The girl refuses to speak in her mother's presence but
eventually admits that she took “many tablets” about 4:00 p.m. (3 hours ago). Which of
the following is recommended as an antidote for this patient's ingestion?
a) Atropine sulfate
b) Hemodialysis
c) Whole bowel irrigation
d) Oral N-acetylcysteine
e) Activated charcoal
43) Which of the following is consistent with abuse rather than accidental injury?
a) A 30-month-old child with a bucket handle fracture
b) A 12-month-old infant with a rib fracture
c) A 6-month-old infant with retinal hemorrhages in the absence of signs of
external head trauma
d) Abdominal bruises in a 9-month-old infant
e) All of the above
44) A 2-year-old presents with painless rectal bleeding. The hemoglobin is 9 g/dL.
Capillary refill remains normal. The best next step to positively identify the cause of
bleeding is:
a) Colonoscopy
b) Transfusion with packed red blood cells
c) Meckel diverticulum scan
d) Gastric lavage
e) Stool culture

DO NOT DISTRIBUTE - 41 -
Pediatrics 01May2009

45) A 3-week-old male infant presents to the emergency department with 24-hour history
of vomiting and poor feeding. He is found to be hypotensive and hypoglycemic. His
serum electrolyte values are as follows: Na 121mmol/L, K 6.9mmol/L, CO2 20mmol/L,
chloride 105mmol/L, BUN 17 mg/dL, creatinine 0.7mg/dL, and glucose 36mg/dL. He
receives 20mL/kg NS fluid bolus and 2mL/kg dextrose 25. What other life-saving
intervention should this infant receive?
a) IV azithromycin
b) IV bicarbonate
c) IV hydrocortisone
d) IV albumin
e) IV calcium
46) A 7-year-old girl presents with a 3-week history of dozens of asymptomatic red, scaly
5 to 10 mm plaques appearing on the trunk. When the scales are pulled off, they bleed.
Her nails are pitted. The most appropriate laboratory test is:
a) A bacterial culture of the red plaques
b) A fungal culture of the red plaques
c) A throat culture
d) A Tzanck smear
e) A complete blood count
47) An 8-year-old patient of yours with attention-deficit/hyperactivity disorder is
experiencing unacceptable adverse effects due to his stimulant medication. You have
prescribed immediate- and extended-release preparations of two separate agents in the
past. You believe that the patient may benefit from switching to a non-stimulant
medication. Which of the following medications approved for the treatment of attention-
deficit/hyperactivity disorder is classified as a non-stimulant?
a) Oral atomoxetine
b) Oral lisdexamfetamine
c) Oral methylphenidate
d) Oral dextroamphetamine
e) Oral mixed amphetamine salts
48) A 9-month-old girl presents with a 3-day history of fever to 103°F (39.4°C). This
morning, the girl developed a rash. On physical examination, the girl is afebrile and has
an erythematous, maculopapular rash over her trunk, arms, and legs. Which of the
following is the most likely cause of this patient's illness?
a) Human parvovirus B19
b) Measles
c) Human herpesvirus 6
d) Chickenpox
e) Group A beta-hemolytic streptococci
49) A 2-year-old child is brought to the emergency department following a brief (<2
minutes) generalized seizure. Initial vitals include a temperature of 102.9°F. Following
the history, physical examination, and laboratory studies, you determine that the patient
has had a febrile seizure. The parents are appropriately concerned and have a number of
questions. You would be correct in telling them which of the following?

DO NOT DISTRIBUTE - 42 -
Pediatrics 01May2009

a) Children who experience a single febrile seizure have no greater risk of


subsequently developing epilepsy than children who have not experienced a
febrile seizure
b) The morbidity and mortality associated with febrile seizures is extremely high
c) At least half of patients who experience an initial febrile seizure will
experience seizures with subsequent episodes of fever
d) Patients who have experienced a single febrile seizure should be placed on
preventative anticonvulsant medication
e) Febrile seizures are usually associated with intracranial infections
50) A 3-year-old boy presents with an elbow hemarthrosis after falling on his elbow.
There is no history of spontaneous bleeding. There is no history of epistaxis, gingival
bleeding, or cutaneous bruising. The child's maternal grandfather had frequent
spontaneous bleeding and hemarthroses after trauma on multiple occasions. Laboratory
results revealed a prolonged PTT, normal PT, and a platelet count of 150,000 per mm3.
The factor VIII coagulant activity (VIII:c) is low and the factor IX level is normal. What
is the most likely diagnosis?
a) Idiopathic thrombocytopenic purpura
b) Von Willebrand disease
c) Vitamin K deficiency
d) Hemophilia A
e) Liver disease
51) A 10-week-old boy is brought to the emergency department by his mother with a
history of failure to thrive and poor feeding. He occasionally vomits small amounts of
formula. His birth weight, length, and head circumference were at the 50th percentile;
however, his weight has dropped to the 10th percentile and his length to between 25th
and 40th percentiles. His vital signs are normal, and the physical exam is otherwise
unrevealing. Venous blood gas and electrolyte study results include: pH 7.32, sodium 134
mEq/L, potassium 4.5 mEq/L, chloride 106 mEq/L, and bicarbonate 10 mEq/L. Which of
the following diagnoses is the most likely?
a) Inborn error of metabolism
b) Renal tubular acidosis
c) Pyloric stenosis
d) Chronic diarrhea
e) Cystic fibrosis
52) A 2-year-old girl presents with a swollen left knee, limping, and morning stiffness in
the left knee of 3 months' duration. On physical examination, there is a left knee joint
effusion, synovial thickening, and limitation of movement. In addition, the left leg is
longer than the right and there is atrophy of the quadriceps. The remainder of the review
of systems and physical examinations is normal. On laboratory testing, a complete blood
count is normal. An antinuclear (ANA) antibody test is positive at a titer of 1:320. This
child is at most risk for which of the following sequelae/complications?
a) Glomerulonephritis
b) Hemolytic anemia
c) Chronic, non-granulomatous anterior uveitis (iridocyclitis)
d) Acute anterior uveitis (iridocyclitis)
e) Rheumatic heart disease

DO NOT DISTRIBUTE - 43 -
Pediatrics 01May2009

53) A 9-year-old boy diagnosed with pneumonia 2 days ago presents to the emergency
department via ambulance in respiratory distress. His past medical history is
noncontributory, and he is at low risk for contracting tuberculosis. He is hypoxic and
requires oxygen. A STAT portable chest radiograph reveals a large right-sided pleural
effusion, which shifts in the decubitus position. Fluid is obtained via thoracentesis for
Gram stain and culture. Which of the following is the most likely pathogen responsible
for this boy's pneumonia?
a) Staphylococcus aureus
b) Nontypeable Haemophilus influenzae
c) Chlamydophila pneumoniae
d) Klebsiella pneumoniae
e) Mycoplasma pneumoniae
54) During a routine annual physical examination, a 9-year-old previously healthy girl
has a blood pressure of 140/75 mm Hg in all four extremities. The physical examination
is otherwise completely normal, except for obesity. The family history is positive for
hypertension in the father and paternal uncle. The blood pressure remains in the 140/70
mm Hg range on two repeat examinations performed 1 week apart, using a cuff that is
appropriate for her obesity. The urinalysis, serum electrolytes, and serum creatinine
levels are normal. Which of the following is the most appropriate next step in the
management of this patient?
a) Reassure the patient that her blood pressure is normal for her size
b) Advise observation, with repeat blood pressure checks every month
c) Advise an immediate evaluation by a nephrologist and cardiologist
d) Advise a regimen of weight reduction and regular exercise
e) Advice a regimen of diuretic therapy
55) A parent brings her 12-week-old child to your office because he has a scaly facial
rash. The boy was exclusively breastfed for 8 weeks but was switched to commercial cow
milk-based formula about a month ago when his mother went back to work. She has been
putting lotion on the rash, but it has not helped. The child's birth weight was at the 50th
percentile but has now dropped toward the 25th percentile line. The physical examination
reveals an eczematous rash over both cheeks. The stool is guaiac-positive but not grossly
bloody. Based on the history and physical examination, you suspect that the patient may
be allergic to cow milk protein. Which of the following is the best next step in the
management of this patient?
a) Recommend that the mother see her obstetrician about medication to help her
begin lactating again
b) Switch the patient from cow milk-based formula to whole cow's milk
c) Switch the patient from cow milk-based formula to soy formula
d) Switch the patient from cow milk-based formula to a protein hydrolysate
formula
e) Begin parenteral alimentation to permit total bowel rest
56) A 16-year-old male is brought to your office by his mother, who insists that you
perform a urine drug screen on her son. You begin by interviewing the mother and the
young man together, but explain to the parent that you will also be conducting part of the
interview and the physical examination without her present in the room. She states that
she will only agree to let you speak with him alone if you agree to discuss with her any

DO NOT DISTRIBUTE - 44 -
Pediatrics 01May2009

high-risk behaviors that he admits to engaging in. Concerning patient confidentiality in


regard to adolescents, you are required by law to inform the parent of this minor of which
of the following?
a) Use of marijuana
b) Suicidal ideation
c) Petty theft
d) Consensual sexual relations with another minor of the opposite gender
e) Consensual sexual relations with another minor of the same gender
57) You see a 4-year-old child for declining school performance and behavior problems.
His mother notes that he is a poor sleeper. He snores loudly and often gasps in his sleep.
Sometimes she sleeps with him because she is afraid he will stop breathing. You note a
slight fall off the growth curve and very large tonsils. A neck film demonstrates large
adenoids as well. The child's insurance company will not pay to have the tonsils and
adenoids removed unless you can prove they are causing him significant health problems.
Which test is the most likely to give you that information?
a) Bronchoscopy
b) Overnight pulse oximetry monitoring
c) Polysomnography
d) Fluoroscopy
e) Overnight EEG monitoring
58) An infant who was discharged from the hospital on day 2 of life presents to your
office 3 days later for follow-up. The mother did not receive prenatal care. You notice
bilateral purulent discharge from the eyes. There is marked eyelid edema and
conjunctival swelling (chemosis). What is the most likely pathologic agent?
a) Chlamydia trachomatis
b) Neisseria gonorrhoeae
c) Group B Streptococcus
d) Toxoplasma gondii
e) Treponema pallidum
59) An unresponsive adolescent patient is brought to the emergency department with
suspected ingestion of an unknown substance. EMS received a call from the hotel room
where the youth was found, but no one else was there when they arrived. The patient is
on 100% inspired oxygen and has required several bouts of positive pressure ventilation
in the ambulance. On exam, the patient has a heart rate of 55, blood pressure 85/50,
pinpoint pupils, and track lines on his left arm. Along with ongoing cardiovascular and
respiratory support, which of the following should be administered to this patient?
a) Pralidoxime chloride
b) Physostigmine
c) Naloxone
d) Atropine sulfate
e) Deferoxamine
60) A 15-month-old boy is brought to the emergency department with a fever and
difficulty breathing. Right-sided wheezing is noted on the physical examination. The
patient does not improve with aerosolized nebulizer treatment. An inspiratory chest
radiograph is normal; however, the expiratory film demonstrates right-sided

DO NOT DISTRIBUTE - 45 -
Pediatrics 01May2009

hyperinflation, with mediastinal shift to the left. This patient's respiratory symptoms are
most likely due to which of the following?
a) Pneumonia
b) Foreign body aspiration
c) Pneumothorax
d) Empyema
e) Viral upper respiratory infection
61) You are seeing an 18-month-old boy who is new to your practice. His father is
concerned about his child's development in relation to his two older brothers. The boy
avoids eye contact and does not respond to efforts to engage him in reciprocal play such
as peek-a-boo and patty cake games. He does not generate spontaneous language but can
repeat certain words if spoken to him over and over. He spends a lot of time by himself
rocking back and forth and becomes very agitated if this activity is interrupted. Which of
the following conditions is most consistent with this child's reported behaviors?
a) Down syndrome
b) Hearing impairment
c) Autism
d) Attention-deficit/hyperactivity disorder
e) Asperger syndrome
62) An 8-year-old boy is referred to the emergency department by his pediatrician for a
chief complaint of weakness. The weakness has been slowly progressive over the last
several weeks. A review of symptoms reveals a history of constipation, polyuria, and
polydipsia. The child is on no medications, and past medical history is noncontributory.
In the primary physician's office, the patient had a serum potassium measurement of 2.8
mEq/L. A blood pressure measurement in the emergency department is normal for age,
height, and gender. Urine electrolyte studies reveal an elevated urine potassium value.
Which of the following conditions is the most likely cause of this patient's hypokalemia?
a) Excessive sweating
b) Renal tubular acidosis
c) Anorexia nervosa
d) Cushing syndrome
e) Renovascular disease
63) A 2-week-old female infant presents with generalized hypotonia, duodenal atresia
and hypothyroidism. What other structural defect is she most likely to have?
a) Malrotation
b) Endocardial cushion defects
c) Cleft palate
d) Renal disease
e) Sensorineural hearing loss
64) Which of the following conditions are often associated with polyhydramnios?
a) Duodenal atresia
b) Tracheoesophageal fistula
c) Congenital hydrocephalus with myelomeningocele
d) Renal agenesis
e) A, B, and C

DO NOT DISTRIBUTE - 46 -
Pediatrics 01May2009

65) A 3-year-old boy presents with violent episodes of intermittent colicky pain, emesis,
and blood per rectum. A tubular mass is palpated in the right lower quadrant. The
abdominal radiograph reveals a dearth of air in the right lower quadrant and air-fluid
levels consistent with ileus. Which of the following procedures will best assist in
diagnosis and treatment?
a) Esophagogastroduodenoscopy
b) Rectal biopsy
c) Air contrast or double contrast enema
d) Stool culture
e) Colonoscopy
66) An 18-month-old female child presents with blood-streaked stool. The stool is
grossly positive on occult blood testing. Which of the following diagnoses is most likely?
a) Anal fissure
b) Peptic ulcer disease
c) Mallory-Weiss tear
d) Inflammatory bowel disease
e) Necrotizing enterocolitis
67) A 4-year-old boy was seen by his pediatrician for fever and abdominal pain. The pain
began after a sledding accident the day before his visit in which he fell on his right side.
His mother noticed that his abdomen appeared distended today, particularly on the right
side. In the pediatrician's office, he is noted to be hypertensive and has gross hematuria.
What is the most likely diagnosis?
a) Pyelonephritis
b) Liver contusion
c) Renal contusion
d) Wilms tumor
e) Neuroblastoma
68) You are called to evaluate a newborn with an apparent foot deformity. On close
examination, you note adduction of the forefoot, inversion of the foot, and plantar flexion
at the ankle that is relatively fixed. Which of the following is true of this patient's
condition?
a) This clinical picture is most consistent with metatarsus adductus
b) This deformity will respond to stretching exercises
c) This deformity will correct spontaneously when the child is able to bear weight
d) This deformity will require surgical repair
e) This deformity may be associated with other congenital malformations
69) A 12-year-old boy with Crohn disease for 2 years is seen with an acute exacerbation.
He is complaining of abdominal pain and diarrhea and has right lower quadrant fullness.
The most effective approach in this acute setting is which of the following?
a) Perform a colonoscopy for cancer surveillance
b) Obtain a stool culture and to exclude acute infectious colitis and imaging
studies to evaluate for abscess or fistula
c) Initiate therapy with mercaptopurine or azathioprine
d) Perform a capsule endoscopy
e) Start biologic therapy with anti-TNF alpha antibody

DO NOT DISTRIBUTE - 47 -
Pediatrics 01May2009

70) A 3-year-old girl periodically experiences swelling around her lips and breaks out in
hives when she eats the snacks provided at daycare. Which of the following is the most
appropriate for determining whether the child's symptoms are due to food allergies?
a) Skin prick testing to foods
b) Food-specific IgE levels
c) Skin prick testing to foods followed by double-blind placebo-controlled food
challenges
d) Open-label food challenges
e) Endoscopy
71) An 11-year-old girl is referred to your office following an abnormal screen for
scoliosis. You diagnose idiopathic scoliosis on exam using Adam's forward bending test.
Subsequent radiographs reveal a lateral curvature of 35-degrees. The patient is pre-
menarchal. You refer the patient to an orthopedic surgeon and counsel the parent that the
specialist will probably recommend:
a) External bracing
b) Follow-up radiographs every 6 months
c) Stretching exercises
d) Surgical fixation
e) No intervention
72) A child in the emergency department has point tenderness over the proximal tibia and
an appropriate history of trauma. The radiograph shows a fracture through the growth
plate that extends into the epiphysis and joint space. This type of fracture would be
characterized as:
a) Salter-Harris Type I
b) Salter-Harris Type II
c) Salter-Harris Type III
d) Salter-Harris Type IV
e) Salter-Harris Type V
73) A 4-year-old Caucasian boy presents for evaluation of persistent jaundice. The family
reports that the boy had neonatal jaundice on the first day of life, and was treated with
phototherapy. He has always had mild icterus, but has had increased icterus at times,
especially following other mild illnesses, such as ear infections and colds. There is a
family history of his father and paternal grandmother having undergone splenectomy. On
examination, the boy has mild scleral icterus, and his spleen is palpable about 3 cm below
the left costal margin. The laboratory evaluation reveals a total bilirubin of 1.9 mg/dL
(unconjugated fraction is 1.5 mg/dL), normal liver transaminases, hemoglobin of 11.2
gm/dL, a normal MCV, and an elevated reticulocyte count of 8%. An osmotic fragility
test is performed and demonstrates positive results. What is the most likely diagnosis?
a) Iron-deficiency anemia
b) Hereditary spherocytosis
c) Acute blood loss
d) Acute leukemia
e) Sickle cell disease
74) An adolescent comes to you with a chief complaint of painless vaginal discharge.
You note projection of the breast areola as a secondary mound above the contour of the
breast and pubic hair of adult texture and color with no spread to the medial surface of

DO NOT DISTRIBUTE - 48 -
Pediatrics 01May2009

the thighs. This patient's examination is most consistent with which Tanner stage of
development?
a) Stage I
b) Stage II
c) Stage III
d) Stage IV
e) Stage V
75) Which of the following statements about acute myeloid leukemia (AML) is true?
a) The preferred treatment for all types of AML is bone marrow transplant
b) Chemotherapy used for AML is more intense than that used for ALL
c) Hyperleukocytosis is not a problem with AML
d) Patients with Down syndrome and AML have a worse prognosis
e) Secondary AML has a good response to therapy
76) A 13-year-old male patient presents with intermittent abdominal pain, diarrhea,
weight loss, and growth failure, and is noted on colonoscopy to have inflammatory skip
lesions throughout the colon with rectal sparing. Which of the following is true?
a) Ulcerative colitis typically is characterized by rectal sparing
b) Ulcerative colitis typically is characterized by skip lesions
c) Crohn disease typically is characterized by transmural disease
d) Ulcerative colitis typically is associated with growth failure
e) Ulcerative colitis typically is associated with perianal disease
77) You are moonlighting in the pediatric emergency department when a 10-year-old
male arrives by ambulance with lethargy, confusion, dizziness, and a severe headache.
His parents and maternal grandmother are in the adult emergency department with less
severe but similar symptoms. The emergency medical technicians report that they were
called by the police who found the family sleeping in their car with the engine running at
their Christmas tree stand. Carbon monoxide poisoning is suspected. Which of the
following should be the first step in the evaluation and management of this patient?
a) Obtain an EKG
b) Draw an arterial blood gas
c) Draw a blood carboxyhemoglobin level
d) Administer 100% oxygen
e) Stabilize the patient for transfer to a hyperbaric oxygen chamber
78) A 5-year-old girl presents to the emergency department with a 12-hour history of
fever and respiratory distress. On physical examination, the girl appears toxic, is
drooling, and leaning forward with her chin extended. She has a temperature of 104°F
(40°C), and a respiratory rate of 32 breaths/minute. Which of the following is the most
likely diagnosis?
a) Epiglottitis
b) Croup
c) Bacterial pneumonia
d) Diphtheria
e) Anaphylaxis
79) In response to your question concerning guns in the home during a routine adolescent
health maintenance visit, the mother of the patient tells you that her husband, the boy's

DO NOT DISTRIBUTE - 49 -
Pediatrics 01May2009

stepfather, keeps a loaded handgun in the bed table drawer for protection. You would be
correct in telling this family that an adolescent who lives in a home with a gun:
a) Is less likely than peers who do not live with guns to die from homicide
b) Is less likely than peers who do not live with guns to commit suicide
c) Is mature enough to use good safety precautions, so storing the handgun
separately from the ammunition is unnecessary
d) Has a 10-fold greater risk of dying from suicide than peers who do not live in
homes with guns
e) Is less likely than peers who do not live with guns to be shot during a domestic
dispute
80) You are seeing a new patient for a health-maintenance visit. The child is able to tell
you his age and gender and speaks in five to eight word sentences. His grandmother tells
you that he is able to pedal a tricycle. He can perform a broad jump when the behavior is
modeled and is able to copy a circle. However, he cannot yet balance on one foot or copy
a cross. You record that the patient's developmental achievement is consistent with his
age. Which of the following most closely correlates with this child's age in years?
a) 2-years
b) 3-years
c) 4-years
d) 5-years
e) 6-years
81) A child weighing 27 kg with a history of vomiting for 36 hours is judged to be 10%
dehydrated based on vital signs and physical examination. The serum sodium
measurement is 134 mEq/L. An initial 540-mL bolus of normal saline results in
stabilization of the heart rate and improved capillary refill. Which of the following is the
most appropriate parenteral fluid choice for the next 8 hours?
a) D5 0.2 normal saline with 20mEq/L KCl (added after urination) at 120mL/hr
b) D5 0.2 normal saline with 20mEq/L KCl (added after urination) at 180mL/hr
c) D5 0.2 normal saline with 20mEq/L KCl (added after urination) at 220mL/hr
d) D 0.45 normal saline with 20mEq/L KCl (added after urination) at 220mL/hr
e) D5 0.45 normal saline with 20mEq/L KCl (added after urination) at 180mL/hr
82) An 8-year-old boy presents with growth failure and vague abdominal pain. The
abdomen is distended. There is no perianal disease, abdominal mass, or tenderness. The
next set of diagnostic tests should include:
a) CBC, CRP, tissue transglutaminase assay
b) CT scan of the abdomen
c) Urinalysis, sweat chloride, laparotomy
d) Colonoscopy, upper endoscopy
e) Stool culture for ova and parasites
83) A 3-year-old boy presents to the pediatrician with fever, pallor, anorexia, joint pain,
petechiae, and hepatosplenomegaly. Which of the following is the most likely diagnosis?
a) Acute lymphoblastic leukemia
b) Acute myelogenous leukemia
c) Juvenile chronic myelogenous leukemia
d) Aplastic anemia
e) Osteosarcoma

DO NOT DISTRIBUTE - 50 -
Pediatrics 01May2009

84) A 16-year-old girl who is 2 years post-menarche presents with mildly uneven
shoulders and a small degree of one-sided rib prominence. Radiographs reveal a 25-
degree scoliosis. Which of the following represents the best treatment?
a) Posterior spinal fusion
b) Intensive physical therapy
c) Scoliosis bracing
d) Spinal manipulation
e) Observation with repeat x-ray in 1 year
85) A 5-year-old boy who returned from a camping trip to his grandparents' farm in
Virginia develops a fever of 103°F, a headache, vomiting, and an erythematous, macular
rash on his wrists and ankles. On physical examination, he is moderately tachycardic with
otherwise stable vital signs and no focal signs of infection. A CBC reveals a normal
WBC count and differential and normal hemoglobin. However, the boy's platelet count is
65,000/mm3. Serum electrolytes are normal. Blood cultures and immunofluorescent
studies are sent. Which of the following is the most appropriate next course of action?
a) Discharge home on amoxicillin with close follow-up and reliable caregivers
b) Discharge home on amoxicillin-clavulanic acid with close follow-up and
reliable caregivers
c) Hospitalization for observation pending further test results
d) Hospitalization for intravenous doxycycline and cefotaxime
e) Hospitalization for intravenous doxycycline
86) A 5-year-old boy presents with painful swelling of the hand and feet since the day
before. Since earlier today, he has palpable purpura on the lower extremities, and also
developed intermittent, colicky midabdominal pain. Prior to these events, he had a cold
for 1 week. He did not have fevers, and overall is well appearing. On physical
examination, he has normal vital signs. He has palpable purpura on the lower extremities
and buttocks. He has scrotal swelling. His hand and feet are puffy, and he has pain with
movement of the ankle joints. His abdominal examination is unremarkable. A complete
blood count shows normal results with a platelet count of 350,000/mm3. Which of the
following laboratory tests is most often abnormal in this disease process?
a) Antinuclear antibody (ANA)
b) Antineutrophil cytoplasmic antibody (ANCA)
c) Complement C3 and C4 levels
d) Urinalysis
e) Serum creatinine
87) A 12-month-old male infant presents with a hemoglobin of 7.5 and a hematocrit of
22%. The mean corpuscular volume is 65 and the adjusted reticulocyte count is 1.0%.
What is the most likely cause of anemia in this child?
a) Iron-deficiency anemia
b) Anemia of chronic disease
c) Transient erythrocytopenia of childhood
d) Thalassemia syndrome
e) Parvovirus B19 aplastic crisis
88) A 12-year-old male adolescent presents with a 1-month history of fever, weight loss,
fatigue, and pain and localized swelling of the mid-proximal femur. Which of the
following is the most likely diagnosis?

DO NOT DISTRIBUTE - 51 -
Pediatrics 01May2009

a) Ewing sarcoma
b) Osteosarcoma
c) Chronic osteomyelitis
d) Benign bone tumor
e) Eosinophilic granuloma
89) You are examining a 3-year-old girl at her well-child visit. While she is staring at her
stuffed cow in your hands, you quickly cover her right eye with an index card. When the
index card is removed seconds later, you notice that the right eye “drifts” back toward the
center. This reaction in response to the cover test indicates what abnormal condition?
a) Strabismus
b) Amblyopia
c) Leukocoria
d) Retinoblastoma
e) Nasolacrimal duct obstruction
90) A 14-year-old girl is brought to your office by her mother because she is complaining
of “seeing double.” The history is significant for headaches that waken the patient from
sleep in the morning but are relieved by vomiting. On physical examination, you note that
she is unable to abduct either eye. Lower extremity reflexes are slightly exaggerated.
Which of the following physical signs is most likely to be present in this patient?
a) Hypotension
b) Papilledema
c) Tachycardia
d) Patency of the anterior fontanelle
e) Erythema migrans
91) A previously healthy 4-year-old girl presents with a history of diarrhea and vomiting
for the past 3 days and decreased urine output for the past 12 hours. On examination, she
has a heart rate of 120 beats per minute, blood pressure of 105/65 mm Hg, and no edema.
The blood tests reveal serum sodium of 128 mEq/L, potassium 5.6 mEq/L, bicarbonate
12 mEq/L, BUN 55 mg/dL, creatinine 1.6 mg/dL. The urine tests reveal a fractional
excretion of sodium of 0.1. The kidney ultrasound is normal. Which of the following
constitutes the most appropriate immediate management of this child's acute renal
failure?
a) Intravenous normal saline bolus to correct the renal hypoperfusion
b) Intravenous bicarbonate to correct the metabolic acidosis
c) Intravenous furosemide to correct the fluid overload
d) Intravenous antibiotics to correct the infectious gastroenteritis
e) Initiation of dialysis to correct the acute renal failure
92) A very tired mother brings her 6-week-old infant to your office because “he screams
for hours and hours a day and nothing makes him stop.” His parent describes the crying
spells as occurring daily and lasting several hours, usually through the late afternoon and
early evening. Nothing seems to console the child during these episodes. While he is
crying, the infant often pulls his knees to his abdomen as if he is in pain. Other than the
crying spells, the child is asymptomatic. He feeds well and moves his bowels regularly.
The child's weight, length, and head circumference are normal, and his physical
examination is normal. This patient's history and physical examination are most
consistent with which of the following conditions?

DO NOT DISTRIBUTE - 52 -
Pediatrics 01May2009

a) Feeding intolerance
b) Cow milk protein allergy
c) Intussusception
d) Hirschsprung disease
e) Colic
93) A newborn infant has a slight hip click on hip examination. Which of the following
risk factors would most strongly support further evaluation?
a) Female patient
b) First born
c) Torticollis
d) Metatarsus adductus
e) Breech presentation or family history of developmental dysplasia of the hip
94) A 14-year-old patient in your practice with anorexia nervosa has fallen to 80% of her
ideal body weight for height and gender. She has not menstruated in 9 months. She has
postural hypotension and a low heart rate. Which of the following murmurs is most likely
to be present on this patient's cardiac examination?
a) A midsystolic click, followed by a murmur
b) A fixed split S2
c) A vibratory holosystolic murmur in both axilla
d) A third heart sound
e) A nonspecific ejection murmur at the base of the heart
95) You are offering preventive counseling to the parent of a 12-month-old child at a
health maintenance visit. The child weighs 18 lbs. You would be correct in informing the
parent that this child should be:
a) Restrained in a rear-facing infant car seat in the back seat of the car until he has
reached 20 lbs. in weight
b) Restrained in a forward-facing infant car seat in the back seat of the car since
he is now #1 year of age
c) Restrained in a rear-facing infant car seat in the front seat of the car until he has
reached 20 lbs. in weight
d) Restrained in a forward-facing infant car seat in the front seat of the car since
he is now #1 year of age
e) Restrained in a forward-facing booster seat in the back seat of the car since he
is now #1 year of age
96) A 4-week-old male infant born at term presents with emesis, dehydration, and poor
weight gain. The pediatrician evaluating the child palpates an olive-sized mass in the
child's epigastrium. She believes the infant may have pyloric stenosis. Which of the
following clinical presentations is most consistent with pyloric stenosis?
a) Projectile nonbilious emesis
b) Bilious emesis
c) Bloody diarrhea
d) Violent episodes of intermittent colicky pain and emesis
e) Right lower quadrant abdominal pain
97) A 5-year-old boy presents to the emergency department with complaints of dizziness
and confusion. Three days before presentation he developed a low-grade fever and
vomited twice. Since then, the fever and vomiting have resolved, but the patient has

DO NOT DISTRIBUTE - 53 -
Pediatrics 01May2009

passed 8 to 10 loose, foul-smelling stools per day. The boy's mother has been afraid to
give him anything but water or diluted juice due to his history of vomiting. Deep tendon
reflexes are diminished throughout. This patient's ataxia and confusion are most likely
due to which of the following electrolyte imbalances?
a) Hypomagnesemia
b) Hyperkalemia
c) Metabolic alkalosis
d) Hypochloremia
e) Hyponatremia
98) A 13-year-old male presents to the office with short stature. Growth data
demonstrates that he has been growing between the 3rd and 5th percentile at a steady rate
since age 4 years. His father started shaving at age 17 and completed his growth at age 19
years. What examination and workup would support the diagnosis of constitutional delay
of growth and puberty?
a) Acne and axillary hair, Tanner III pubic hair, testicular volume 12 cc, bone age
14 years, TSH 1.5 (0.5 to 4.8), IGF-I 340 (152 to 540)
b) No axillary hair, Tanner I pubic hair, testicular volume 4 cc, bone age 11 years,
TSH 12 (0.5 to 4.8), IGF-I 200 (152 to 540)
c) Scant axillary hair, Tanner II pubic hair, testicular volume 5 cc, bone age 11
years, TSH 2.1 (0.5 to 4.8), IGF-I 420 (152 to 540)
d) No axillary hair, Tanner I pubic hair, testicular volume 4 cc, bone age 11 years,
TSH 3.1 (0.5 to 4.8), IGF-I 62 (152 to 540)
99) A 24-month-old male in your office for his regular health maintenance visit has the
following results on screening tests: hemoglobin 9.6 g/dL; capillary blood lead level 16
mcg/dL. He lives in Section 8 housing in poor repair built before 1960. Which of the
following is the most appropriate next course of action?
a) Counsel the family regarding lead removal and recheck the level in 6 months
b) Refer the family to the local governmental lead management agency
c) Obtain a venous lead level for confirmation
d) Start the patient on oral succimer on an outpatient basis
e) Obtain neurodevelopmental testing for the patient
100) A 10-year-old girl presents with a linear streaks of thickened and indurated skin on
the right arm and trunk. The linear streak on the right arm has a longitudinal orientation
and extends from the upper arm to the dorsal aspect of the hand, whereas the linear streak
on the trunk is transversely oriented. The lesions are surrounded by a halo of erythema
with a violaceous appearance. The central portion is hyperpigmented and thickened.
Which of the following complications is this child most likely to develop?
a) Esophageal dysfunction
b) Pulmonary fibrosis
c) Contracture of the right elbow
d) Raynaud phenomenon
e) Digital necrosis

James Lamberg

DO NOT DISTRIBUTE - 54 -
Pediatrics 01May2009

AnswerKey 45) C 23) E Peds #3


Peds #1 46) E 24) B 1) C
1) A 47) D 25) A 2) D
2) C 48) C 26) C 3) A
3) E 49) F 27) D 4) A
4) B 50) A 28) F 5) C
5) D 51) B 29) C 6) C
6) A 52) C 30) C 7) E
7) E 53) E 31) B 8) D
8) B 54) E 32) A 9) D
9) D 55) D 33) D 10) B
10) C 56) A 34) E 11) C
11) B 57) E 35) D 12) C
12) A 58) B 36) C 13) B
13) E 59) A 37) C 14) A
14) C 60) D 38) C 15) B
15) C 61) C 39) D 16) D
16) A 62) D 40) E 17) A
17) A 63) C 41) D 18) D
18) E 64) D 42) B 19) D
19) A 65) C 43) E 20) C
20) B 66) A 44) D 21) D
21) D 45) D 22) E
22) F Peds #2 46) D 23) A
23) D 1) C 47) B 24) D
24) D 2) D 48) D 25) A
25) B 3) C 49) A 26) B
26) B 4) B 50) C 27) E
27) E 5) B 51) C 28) A
28) B 6) A 52) A 29) C
29) A 7) C 53) E 30) A
30) A 8) B 54) C 31) D
31) C 9) E 55) E 32) C
32) D 10) D 56) D 33) B
33) D 11) C 57) A 34) D
34) A 12) B 58) A 35) B
35) D 13) E 59) E 36) A
36) C 14) B 60) C 37) E
37) C 15) E 61) D 38) C
38) D 16) D 62) A 39) A
39) B 17) A 63) B 40) A
40) B 18) C 64) E 41) B
41) D 19) B 65) D 42) B
42) E 20) A 66) A 43) C
43) A 21) C 44) D
44) F 22) D 45) A

DO NOT DISTRIBUTE - 55 -
Pediatrics 01May2009

46) D Peds #5 46) C 92) E


47) E 1) B 47) A 93) E
48) D 2) A 48) C 94) A
49) C 3) D 49) C 95) A
50) B 4) D 50) D 96) A
51) C 5) A 51) A 97) E
52) D 6) B 52) C 98) C
53) A 7) A 53) A 99) C
54) E 8) C 54) D 100) C
55) C 9) A 55) D
56) E 10) A 56) B
57) D 11) C 57) C
58) A 12) D 58) B
59) C 13) B 59) C
60) C 14) B 60) B
61) A 15) B 61) C
62) E 16) E 62) B
63) C 17) C 63) B
64) E 18) A 64) E
65) C 19) A 65) C
66) D 20) B 66) A
67) E 21) C 67) D
68) D 22) D 68) E
23) A 69) B
Peds #4 24) B 70) C
1) D 25) B 71) A
2) D 26) D 72) C
3) B 27) A 73) B
4) C 28) C 74) D
5) E 29) C 75) B
6) A 30) E 76) C
7) A 31) D 77) D
8) B 32) A 78) A
9) C 33) A 79) D
10) E 34) D 80) B
11) A 35) B 81) C
12) D 36) E 82) A
13) C 37) C 83) A
14) A 38) C 84) E
15) B 39) D 85) D
16) B 40) D 86) D
17) D 41) E 87) A
18) C 42) D 88) A
19) D 43) E 89) A
44) C 90) B
45) C 91) A

DO NOT DISTRIBUTE - 56 -

You might also like